[ /tv/ /rf/ /vg/ /a/ /b/ /u/ /bo/ /fur/ /to/ /dt/ /cp/ /oe/ /bg/ /ve/ /r/ /mad/ /d/ /mu/ /cr/ /di/ /sw/ /hr/ /wh/ /lor/ /s/ /hau/ /slow/ /gf/ /vn/ /w/ /ma/ /azu/ /wn/ ] [ Main | Settings | Bookmarks | Music Player ]

No.74216 Reply
File: пруф.jpg
Jpg, 86.12 KB, 507×398 - Click the image to expand
edit Find source with google Find source with iqdb
пруф.jpg
Кафедра математики упорно продолжает свою работу. Обсуждаем математику, помогаем с интересными домашними заданиями, решаем то, что не осилил ВольфрамАльфа, популяризируем ТеХ.

Полезные ссылки:
Калькулятор и редактор формул: http://ru.numberempire.com/texequationeditor/equationeditor.php
Образцы разметки: enwiki://Help:Formula
Вольфрамальфа: http://www.wolframalpha.com/
LaTeX pastebin: http://mathbin.net
Математический справочник: http://dxdy.ru/matematicheskij-spravochnik-f52.html

Предыдущий: >>66948
>> No.74222 Reply
Аноны, предлагаю сделать актуальную программу математического обучения. Она, по моей мысли, должна быть трёхуровневой. Первый уровень - вводная часть, понятная даже девятикласснику. Второй - математика, входящая в программу современных нам российских университетов. Третий должен позволить понимать статьи, размещённые на arxiv.org.

Если вы хотите помочь, то предлагайте полезные книги, ссылки на библиотеки и торренты. Пожелания и предложения приветствуются.

Программы, которые показались мне интересными:
http://www.math.harvard.edu/graduate/index.html#qualifying - Гарвард
http://imperium.lenin.ru/~verbit/MATH/programma.html - хоть Вербицкий и упорот, мысли у него дельные.

Реквестирую ссылки на стандарт ЕГЭ по математике и стандарты итоговой госаттестации.
>> No.74256 Reply
>>74222
Предлагаю в первую(вторую) часть включить "Конкретную математику", или создать еще один уровень "для практикующих" куда включить основы мат.анализа/линейной алгебры, и то что еще может пригодится при более менее простых расчетах.
А вообще я не шарю в хороших книгах
>> No.74261 Reply
>>74222
Посмотрел программу Вербицкого. Он игнорирует логическую структуру и гоняется за вызубриванием энциклопедий.
Думаю, корень нашего расхождения здесь:
> я уверен, что самоценности математика сама по себе не имеет. Иначе математика оказывается своего рода сложной интеллектуальной игрой
- говорит он.
>> No.74267 Reply
Оказалось, все просто.
Читаем тут: ruwiki://Математика
Подзаголовок "Разделы математики". Открываем каждую ссылку (матанализ, теорвер и т.п.) и внизу видим список использованной литературы. Изучаем все по порядку. Профит.
Как-то так.
>> No.74273 Reply
File: 0303109v1.pdf
Pdf, 0.26 KB, 612×792 - Click the image to get file
0303109v1.pdf
>>74256
> Предлагаю в первую(вторую) часть включить "Конкретную математику"
Разумно. Можно даже сделать курс "computer science", куда включить комбинаторику, булеву алгебру, логику нулевого, первого и второго порядков, теорию типов, автоматы и игры (я ничего не забыл)?
> куда включить основы мат.анализа/линейной алгебры
Конечно включим. Наш курс математики должен быть актуальным, следовательно, должен давать возможность изучившим половинку первого уровня сдать ЕГЭ на сто баллов, изучившим второй уровень - свободно читать учебники физики и пройти итоговую аттестацию в универе на отлично. Изучившим третий - понимать без подготовки файлы навроде приложенного.
У тебя есть какие-нибудь пожелания? Возможно, составишь список тем, которые тебе интересны?

>>74267
Читать мегабайты книг, не зная заранее их ценность, малополезно. Нужно таки сделать список годноты.
Кстати, вот ссылка на интересное сообщество: http://eek.diary.ru/

>>74261
Математики, которая не может быть к чему-нибудь приложена, вообще быть не может. Например, вся физика - это математика, которой описывается некая модель окружающего нас мира. Вот байка в тему с dxdy:
Говорят, что как-то один диссертант представил диссертацию под названием типа: "Циркуляция ротора кусочно гладкой функции по недиффиренцируемому многообразию". Защита провалилась, так как в то время все диссертации рассматривались в плане их приложений к народному хозяйству. Предложили переработать в плане этих самых приложений. Диссертант долго не думал, и переделывать много не стал, в тексте не изменил ни одной буквы, изменил лишь название. Повторная защита прошла на ура. Теперь она называлась так: "Качение сучковатого бревна по шероховатой поверхности".

Вот ещё копипаста за авторством Арнольда, воззрения которого я разделяю:
http://ega-math.narod.ru/Arnold2.htm
>> No.74277 Reply
>>74273
> Читать мегабайты книг, не зная заранее их ценность, малополезно. Нужно таки сделать список годноты.
Тогда давай, начнем.
Математический анализ - Фихтенгольц, 3 тома. Думаю, этого вполне достаточно.
Алгебра - ?
Аналитическая геометрия - ?
Линейная алгебра и геометрия - ?
Дискретная математика - ?
Математическая логика - ?
Дифференциальные уравнения - ?
Дифференциальная геометрия - ?
Топология - ?
Функциональный анализ и интегральные уравнения - ?
Теория функций комплексного переменного - ?
Уравнения в частных производных (вместо этого курса физикам читаются Методы математической физики) - ?
Теория вероятностей - ?
Математическая статистика - ?
Теория случайных процессов - ?
Вариационное исчисление и методы оптимизации - ?
Методы вычислений, то есть численные методы - ?
Теория чисел - ?
>> No.74280 Reply
>>74273
> (я ничего не забыл)?
Если ты хотел перечислить науки, входящие в CS двадцатого века, то нет.
> сдать ЕГЭ на сто баллов
> свободно читать учебники физики
> понимать без подготовки файлы навроде приложенного
Я уверен, ты понимаешь, что эти задачи никак не связаны между собой. То есть они осуществляются не последовательно, а существуют независимо друг от друга.

>>74222
Обе ссылки прекрасные просто. С удовольствием порешиваю кволовские задачки.
>> No.74283 Reply
>>74277
> Фихтенгольц, 3 тома. Думаю, этого вполне достаточно.
Не достаточно, совсем нет. Фихтенгольц неполноценен. Моё предложение: Зорич, затем Гурса - действительный анализ, Шабат - комплексный анализ.
> Линейная алгебра
Что лучше: Ильин-Позняк, Головина, Беклемишев или Ефимов-Розендорн?

>>74280
> Я уверен, ты понимаешь, что эти задачи никак не связаны между собой
Связь прослеживается, но она слаба, поэтому я и предлагаю разбить программу на три уровня.
> Если ты хотел перечислить науки, входящие в CS двадцатого века, то нет.
Ок. Предлагай уровень двадцать первого века.
>> No.74285 Reply
>>74283
> уровень двадцать первого века
+ Теория категорий (для программистов)
+ Дифференциальная/алгебраическая геометрия/топология, т.е. в сумме 4 науки (для data analysis)
> Что лучше: Ильин-Позняк, Головина, Беклемишев или Ефимов-Розендорн?
Лучше, конечно же, Кострикин-Манин
>> No.74287 Reply
>>74277
Можно добавить "Теорию операторов" Садовничего. В своё время с удовольствием прочитал.
>> No.74306 Reply
File: Майлз_Рид-Алгебраическая_геометрия_для_няшек.pdf
Pdf, 1.54 KB, 595×842 - Click the image to get file
Майлз_Рид-Алгебраическая_геометрия_для_няшек.pdf
Подходите сюда, берите книжечки, не стесняйтесь.
Майлз Рид, "Алгебраическая геометрия для няшек"
>> No.74309 Reply
>>74285
+ Теория категорий (для программистов)
Что можно почитать по теме? Именно не про саму теорию категорий, а про ее связь/приложения в CS.
>> No.74310 Reply
File: 1340651391880.png
Png, 0.73 KB, 300×20 - Click the image to expand
edit Find source with google Find source with iqdb
1340651391880.png
>>74273
> Математики, которая не может быть к чему-нибудь приложена, вообще быть не может.
Как знаешь. Я-то говорил не о том, может быть или не может, а о том, что смотрю на неё иначе: для меня важна логическая система, а не катание брёвен.
>> No.74313 Reply
У меня есть излюбленная тема, которую я придумал, и даже знаю,
как про нее прочитать лекцию, но никогда этого не делал. Называется она «Почему дважды два –– всегда четыре». В этом сюжете очень важное слово –– это «всегда». Почему дважды два –– четыре, можно продемонстрировать хоть на столе: взять два кусочка мела и еще два. Но вот почему всегда четыре?
Развиваться эта тема должна так. Раз «всегда», значит, речь идет о том, что тут запрятан какой-то закон сохранения. Закон сохранения –– это идея, конечно, не математическая; это идея физическая. Простейший вид закона сохранения––это нечто, что аддитивно по областям пространства-времени. Значит, надо уже говорить не о пространстве, а о пространстве-времени, потому что слово «всегда» присутствует. Значит, у вас имеется 4-мерное (а может, какое-нибудь еще) многообразие––пространство-время; вы воображаете, что его можно разбивать на области, в каждой из этих областей что-то такое содержится; и вот, когда вы прибавляете одну область к другой, то, что там содержится, должно прибавляться. Вы узнаёте лагранжиан? Если есть аддитивность, значит, есть лагранжиан, действие и т. д.
Таким образом от сюжета «а почему дважды два всегда четыре» вы
подымаетесь довольно быстро наверх и попадаете в область гораздо более сложную, чем первоначально рассчитывали. Обычно, дойдя до лагранжиана, я в воображении сам себя останавливал и решал, что, пожалуй, для general audience дальше будет трудно продолжать.
>> No.74314 Reply
Вброшу свои 5 копеек. Мыслей много, поэтому будут тезисы:

Теорминимум вербятки ужасен.
Много писал, потом забил. Кто знает, тот знает, а кто не согласен, так я его могу спросить сейчас по этому теорминимуму пару вопросов, что он мне только через неделю нагуглит ответ. И возникнет вопрос: зачем проповедовать то, что сам не практикуешь?

Теорминимум для физиков обязателен
В него входит, конечно, сначала Арнольд "классическая механика", потом алгебры и группы Ли, потом с миру по нитки по чуть-чуть со всех областей в математике. В принципе, достаточно отходить на ранних курсах стекляшку, фпфф или локальные курсы в МФТИ, на физфаке. И вообще у физиков совсем другой подход к математике изначально. Сейчас же речь не об этом.

Подготовить анона до уровня специалиста невозможно.
Речь не о том, что без рашкинского образования нельзя заниматься математикой. Можно. Есть примеры людей, которые уезжали из каких-то там томских пту на phd и быстро все осваивали. Но они осваивали математику в кругу себе подобных. Наша борда с этим не справится. Это неподъемная задача. Мне, например, скучно объяснять, обсуждать математику в общем, тем более в течение долгого времени. В школе преподавать мне кстати нравится, но на доброчане это будет кушать слишком много времени. И я не вижу профита для себя.

Конфликт авторитетов
Допустим, вы все же решили прокачать анончика. Но как он определит, старшекурсник вы или только хорошо маскируетесь? То есть на словах вы Лев толстой, а на деле 80% задач тривиума Арнольда вам не под силу. Причем вы с пеной у рта будете еще ему доказывать, что необязательно вам сейчас знать, как берется какой-то там комплексный интеграл, потому что машинерия не математика. Вопрос открытый.

Теперь предложение.
кафедра-тред с домашним заданием
Возьмем, например, область, которая мне интересна: это
http://arxiv.org/archive/q-fin
Quantitative Finance
В принципе, конкретно для этой области я представляю, как должна выглядеть программа.
http://iafe.org/html/resources_core.php
Я знаю, наверное, 30% этой программы. Соответственно, лично для себя я буду составлять программу занятий, чтобы все осилить и сдать соответсвующий сертификационный экзамен. Это моя личная мотивация. Я думаю, если есть заинтересованные в банковском деле анонимусы с физическим и математическим бэкграундом, мы могли бы поднять хороший тред. Причем, чтобы вникать в q-fin хватает, на самом деле, 3 курсов универа. И то, наверное, уравнения в частных производных можно как-то просто объяснить и младшекурсникам. Чтобы они их сейчас узнали и не боялись. Алсо, так как эта область в рашке развивается всего лишь группой Ширяева на мехмате, то я укажу, какие ништяки входят в программу.
> Stochastic calculus
Теория меры, формула Ито.
> PDEs applied to finance
Уравнения математической физики же.
> Numerical methods
Скучней вычматов я предмета что-то не помню, но это совковая школа. Мы же можем преподнести его в разу интересней.
> Мат. статистика, теория игр, Time series analysis, теория оптимизации + куча вещей из computer science
Так вот, допустим, есть у нас Анон, который хочет въехать в матан. Очень-очень хочет. Анон у нас прогер. Лекции он продалбывал, но все-таки что-то отложилось у него в голове. У анона есть работа, но он готов, допустим, по часу в день выделять на матан. Или два выходных по 2+2 часа. + он готов по скайпу пообщаться на тему того, что же он знает. Или решить высланные по почте простые задачи. Время решения неограничено. Потом анон нам их присылает, становится видно, какой у него уровень. Далее, анону предлагается либо вкурить одному фихтенгольца\зорича, потому что ну никто не будет тратить время на объяснение того, что такое предел. Либо предлагается въехать в предложенные уроки в соответсвующем его натуре треде. То есть я готов запилить тред и поддерживать по теме q-fin, еще я видел людей, которые метят в hep-th. В треде собственно, можно просто излагать последовательно основные моменты какой-то книжки и самые приятные, на ваш вкус, упражнения. Если люди будут примерно с одинаковой скорость читать заданный учебник, то даже будет актуальное обсуждение. Как-то так. В общем, я ратую за сектантский подход к математике. Запиливаем секту алгебраистов и вербуем прихожан. Привет.
>> No.74319 Reply
>>74277
Очень годное начинание, нужно потом список оформленный выложить, как во многих других добротредах u.
Алгебра (изи левел) - Курош, "Курс Высшей Алгебры"
Дискретная математика - Rosen, "Discrete Mathematics and Its Applications" (насколько я знаю, на русском нет)
>> No.74323 Reply
> 74309
Блядь, не говори херни. Если ты хочешь почитать "про приложения", то придётся сделать 2 вещи:
1. Соснуть хуйца.
2. Идти читать статьи про Хаскель и трепаться в /s/.
Причём первый пункт обязателен, потому что никаких "приложений" несмотря на создание иллюзий этого - нету. Просто понимание или есть, и его нет. Это и есть, - фу, щас стошнит - приложение.
>> No.74332 Reply
Ворчун в треде -->>74314
Все в кабриолет!
Финансовый "математик" -- унтерменш, смейтесь над ним, унижайте его, ну вы понели
Тривиум Арнольда не нужен, лол
>> No.74337 Reply
>>74332
> Тривиум Арнольда не нужен, лол
хоти@не смоги
>> No.74339 Reply
>>74337
А ты его видел? А предисловие читал? lol'd
Это тривиум для физиков.
То, что хорошо для физиков, инженеров, бухгалтеров и т.д. не нужно математикам.
>> No.74340 Reply
File: Lawrence-portrait-Gentleman-medium.jpg
Jpg, 106.68 KB, 318×400 - Click the image to expand
edit Find source with google Find source with iqdb
Lawrence-portrait-Gentleman-medium.jpg
>>74332
> Тривиум Арнольда не нужен, лол
Извольте либо обосновать, либо сей секунд отбыть.
>> No.74342 Reply
File: Арнольд-Тривиум.pdf
Pdf, 0.44 KB, 461×673 - Click the image to get file
Арнольд-Тривиум.pdf
Сабж.
>> No.74344 Reply
>>74340
ну, физикам оно, может, и нужно. Я ж не студент-физик.
См -->>74339
>> No.74345 Reply
> > All
Ух вы мои лапочки! Давно завидую людям с увлечённостью математикой. Сам могу выучить только то, что тут же нужно применить к практике, беда прямо.
Всё сказал.
>> No.74347 Reply
>>74344
Ты сам-то его читал, няша? Слабо хотя бы половину сделать?
>> No.74350 Reply
>>74344
> Студент, которому для вычисления с десятипроцентной точностью среднего от сотой степени синуса требуется значительно больше пяти минут, не владеет математикой, даже если он занимался нестандартным анализом, универсальными алгебрами, супермногообразиями или теоремами вложения.
>> No.74351 Reply
>>74350
я сам и мои преподаватели принадлежат той математической школе, в рамках которой это утверждение носит скорее юмористический (можно сказать мягче: эпатажный) характер
>> No.74352 Reply
File: macro-такие-дела.png
Png, 147.43 KB, 340×255 - Click the image to expand
edit Find source with google Find source with iqdb
macro-такие-дела.png
>>74351
В таком случае ваша школа не имеет права называться математической.
>> No.74353 Reply
>>74352
я уже писал раза два в этих тредах, кто в рамках школы, в которой я обучаюсь, называется математиком:
человек, который в состоянии без подготовки понять большую часть статей из основных математических разделов Архива.
>> No.74354 Reply
File: Хома-и-суслик-1.jpg
Jpg, 33.14 KB, 512×384 - Click the image to expand
edit Find source with google Find source with iqdb
Хома-и-суслик-1.jpg
>>74353
Видите ли, сударь, понимание Архива есть всего лишь понимание Архива, не больше. Если вы не можете справиться с "Тривиумом", ваше образование несовершенно, и вам, по-хорошему, надлежит усовершенствовать себя. Вы же упорно пытаетесь преподнести невежество как что-то хорошее, уподобляясь тем самым людям с ГСМ. Я вновь заявляю, что знание лучше, чем незнание.
>> No.74355 Reply
>>74354
просто у тебя дилетантское понимание математики
Математик это не тот, кто умеет решать любые задачки с интегралами, лол
>> No.74356 Reply
>>74355
Умение решать "задачки с интегралами" является необходимым признаком математика.
>> No.74357 Reply
>>74356
> Умение решать "задачки с интегралами" является необходимым признаком математика
не удивлюсь, если на каком-нибудь мехмате действительно так и учат
>> No.74358 Reply
>>74314
> из каких-то там томских пту
Не ПТУ, а ТПУ блджад.
> Допустим, вы все же решили прокачать анончика
Естественно, заниматься бесплатным репетиторством скучно. Нужно просто составить список тем и список литературы по каждому из уровней >>74222.
> В общем, я ратую за сектантский подход к математике
Тру-сектанство на доброчане не взлетит, секту с блекджеком выгоднее организовывать на dxdy.
> И то, наверное, уравнения в частных производных можно как-то просто объяснить и младшекурсникам.
Зачем "как-то"? "Младшекурсник" не то же самое, что и "идиот".

Предлагаю добавить в программу книжку Успенского "Апология математики". Она интересная.
>> No.74359 Reply
>>74357
Ок. Спрошу в лоб. "Тривиум" решишь?
>> No.74360 Reply
>>74222
Аноны, составьте список учебников, по которым вы учились?
>> No.74361 Reply
>>74359
Конечно же нет кроме геометрических задач
Это же упражнения для физиков. Зачёт по бухгалтерскому учёту, например, я тоже не сдам.
>> No.74362 Reply
>>74361
Что же ты тогда умеешь?
>> No.74364 Reply
>>74362
Смотря с чем сравнивать.
Если с тем, что мне предстоит постичь, то почти ничего.
Если с окружающими, то с вероятностью... ох, ну, не буду же я себя расхваливать.
>> No.74365 Reply
>>74364
А ты прямо списочком оформи.
>> No.74366 Reply
>>74362
Я думаю тебе стоит прекратить с ним спорить, на самом деле. Он умеет понимать небольшой круг статей в области, которой занимается его научник. Может быть, даже в рамках данных ему знаний что-то решить может. Ему этого хватает. Ко всему остальному испытывает отвращение, дескать, "это не математика, а физика\бухгалтерское дело\...". Только то, чем он занимается - это математика. Доказывается это тем, что препринты его научника лежат в архиве. А то, что h-index научника не больше трех, так этот H-index придумали физики и вообще мы математики, а не физики. То есть я могу бесконечно продолжать подобную совковую логику.

Мне даже понятно, откуда она берется. В нормальных вузах преподаватели заставляют решать на младших курсах тонны задач. Вуз = говно, следовательно, решать задачи не нужно. Да и вообще, в его понимании, задача с численным ответом - это обязательно интеграл. И так далее. Сколько раз уже это видел.
>> No.74367 Reply
>>74365
Мы перешли от обсуждения тривиума к обсуждению моего отношения к тривиуму, но почему ты подумал, что мне хочется обсуждать свою математику с человеком, которого я заведомо считаю некомпетентным?
>> No.74369 Reply
File: 397px-John_Forbes_Nash,_Jr._by_Peter_Badge.jpg
Jpg, 51.41 KB, 397×599 - Click the image to expand
edit Find source with google Find source with iqdb
397px-John_Forbes_Nash,_Jr._by_Peter_Badge.jpg
>>74367
Потому что у тебя какая-то своя, особенная математика. Хотелось бы узнать о ней побольше. Не отделывался ссылками на Архив, пожалуйста, а просто и по-русски/английски расскажи, что именно ты умеешь.
>> No.74372 Reply
>>74369
> Хотелось бы узнать о ней побольше
Так бы сразу и сказал. Узнать о ней очень просто.
enwiki://Algebraic_topology
enwiki://Differential_topology
enwiki://Differential_geometry
enwiki://Symplectic_geometry
enwiki://Algebraic_geometry
Для начала.
> что именно ты умеешь
> математика. Хотелось бы узнать о ней побольше
Так ты уж определись, ты хочешь поговорить обо мне или о математике?
Я (как и любой), конечно, люблю поболтать о себе, лол. Но зачем это тебе? Ты преследуешь какие-то интимные цели?
>> No.74380 Reply
>>74323
Меня, как всегда, не так поняли, хочется прочитать про CS-направленную теорию категорий. И откуда столько боли от слова приложение
>> No.74381 Reply
>>74372
И при этом ты не знаешь действительный анализ? Странно.
>> No.74382 Reply
>>74380
Маклейна книжка очень хорошая. А если это не то, что тебе нужно, то гугл много чего подобного выдаёт.
>> No.74383 Reply
>>74372
> Я (как и любой), конечно, люблю поболтать о себе, лол.
А я продублирую его просьбу.
Причём, прошу заметить, что делаю это не для наездов, ибо сам я далеко не математик, а стремящийся, лол, и большая часть этих ваших занятий для меня - китайская грамота. Даже с интегралами я справляюсь хуёво, не говоря уже о глубоко теоретических, так сказать, разделах. Хочу стать грамотным человеком, и мне хотелось бы себе сказать, что за ближайший год, например, хочу научиться... научиться чему? Я не вполне знаю. Есть, конечно, вещи, которые мне нужны для понимания каких-то конкретных областей (ML, AI, CV), но это другое дело. Что же касается, как ты говоришь, математики, я не знаю, куда я иду, читая что-то. То есть, критерий "Тривиума" мне понятен (я, конечно, не знаю, что там за задачи и кто из вас прав, сейчас меня не это волнует; алсо, да, Арнольда не уважаю, но это скорее субъективное "потому что он мудак", а не что-то по делу), я беру книжку и либо решаю там всё, либо нет. Всё просто.
Ты, ответив на вопрос "что именно ты умеешь", мог бы сделать для меня доброе дело, дав мне ещё один критерий оценки знания. Это для меня важно, потому что в конечном счёте всё оценивается именно умением, способностью сделать что-то, что я не могу сейчас, и важно помнить об этом при чтении чего-то. Подобно тому, как просмотр фильмов с Джеки Чаном не делает меня "знающим кунг-фу", чтение (и, частично, даже понимание) книг по какой дисциплине не делает меня умеющим что-то.

Конечно, я был бы благодарен, если бы твой оппонент также ответил на мой вопрос, опустив часть с "Тривиумом", потому что про него, как сказано, и так всё ясно.
>> No.74386 Reply
>>74380
Букур, Деляну. "Введение в теорию категорий и функторов"
http://rghost.ru/39875791

Приложения:
http://habrahabr.ru/post/125782/
http://habrahabr.ru/post/133277/
>> No.74388 Reply
>>74381
Да ты охуел.
Я очень люблю анализ, но какое отношение имеет к анализу взятие интегралов?
(есличо, это риторический вопрос)
>>74383
> Арнольда не уважаю, но это скорее субъективное "потому что он мудак"
Да ты точно охуел! Арнольд был один из сильнейших и остроумнейших математиков современности. Его не всегда легко понять, я его книжечки уже года три регулярно перечитываю и только сейчас начал постигать. А в "мат.методы классической механики" вообще можно найти ответ на любой вопрос мироздания, лол.
> ещё один критерий оценки знания
Критерий оценки понимания конкретного предмета -- умение за разумное время прорешать экзамен, составленный компетентным человеком. На сайте НМУ, например, у большинства курсов выложены задачки итогового экзамена.
>> No.74389 Reply
>>74388
> Критерий оценки понимания
Просто "критерий понимания"
fixed
>> No.74392 Reply
ftp://ftp.mccme.ru/users/prasolov/geometry/book.pdf - отличное введение в геометрию. Поймёт даже школьник. При чтении может потребоваться словарик.
>> No.74405 Reply
>>74388
> Я очень люблю анализ, но какое отношение имеет к анализу взятие интегралов?
Ох-ох.
>> No.74408 Reply
>>74388
> но какое отношение имеет к анализу взятие интегралов?(есличо, это риторический вопрос)
Интеграл Лебега, криволинейные/контурные интегралы, поверхностное интегрирование векторных полей/дифференциальных форм, и всё в топку. Ну удачи вам.
>> No.74411 Reply
>>74339
> То, что хорошо для физиков, инженеров, бухгалтеров и т.д. не нужно математикам.
Апачую.
>> No.74418 Reply
>>74388
> Я очень люблю анализ, но какое отношение имеет к анализу взятие интегралов?
Позвольте, а что ж тогда анализ?
>> No.74434 Reply
>>74342
А чем, простите, эти задачи сложнее задач из обычных задачников?
>> No.74437 Reply
>>74408
У тебя проблемы с reading comprehension?
Какое отношение к перечисленному имеют трюки по выражению первообразных одних элементарных функций через другие элементарные функции?
(вообще говоря, это тоже риторический вопрос)
>>74418
Я бы поискал обзорную статью, где сильнейший из ныне живущих аналитиков подробно объясняет, что есть математический анализ, но, мне кажется, что тебе в действительности хочется не приобщиться к науке, а просто поспорить.
>> No.74438 Reply
>>74222
Очень прошу дать конкретные советы по подготовке к поступлению в вуз. Что конкретно нужно сделать человеку, который математику не учил после окончания школы года три-четыре, чтобы полностью восстановить знания по школьной программе и успешно поступить? Вуз чешский, так что ЕГЭ не имеет значения. Вопрос больше по: учебникам, последовательности, времени изучения и так далее.
>> No.74440 Reply
File: 134540723232604.jpg
Jpg, 17.09 KB, 476×95 - Click the image to expand
edit Find source with google Find source with iqdb
134540723232604.jpg
>>74438
Как ты знаешь, в средней школе математика делится на "алгебру и анализ" и "геометрию".
Для изучения первого советую использовать учебники Виленкина, Ивашева-Мусатова, Шварцбурга за 10-11 кл. На это может уйти месяц или даже 3-4 месяца, смотря как сам определишь сроки.
А вообще вдобавок к этому посоветовал бы "Основы анализа" Эдмунда Ландау, так как на мой взгляд школьный курс обрывочен и эта книга позволяет уяснить многие вопросы, при том что доступно написана. Правда, там вряд ли говорится о многочленах, уравнениях и неравенствах.
Источник задач не знаю. Можно из учебника Виленкина, Ивашева-Мусатова, Шварцбурга брать.
Для изучения второго сгодятся книги Атанасяна "Геометрия 7-9" и "Геометрия 10-11". Это может занять от полутора до 5-6 месяцев.
>> No.74443 Reply
File: 1250219469377.jpg
Jpg, 202.12 KB, 400×400
Your censorship settings forbid this file.
unrated
>>74437
> какое отношение имеет к анализу взятие интегралов?
> Какое отношение к перечисленному имеют трюки по выражению первообразных одних элементарных функций через другие элементарные функции?
Жаль нельзя человека принудительно переквалифицировать из математика, ну не знаю, в юриста. Там как раз ценятся подобные виляния.
>> No.74444 Reply
>>74443
Ви что, тупой?
Взятие интегралов := трюки по выражению первообразных одних элементарных функций через другие элементарные функции
>> No.74447 Reply
>>74444
Да, с юристом я видимо перегнул.
>> No.74456 Reply
>>74444
> трюки по выражению первообразных одних элементарных функций через другие
Но в тривиуме нет таких заданий. "В лоб" там ничего не решается.
>> No.74459 Reply
>>74444
Ты ведь умеешь складывать натуральные числа? Или тоже не нужно?
>> No.74460 Reply
>>74440
> А вообще вдобавок к этому посоветовал бы "Основы анализа" Эдмунда Ландау
Это не нужно школьнику, совсем не нужно. Книга Ландау интересна только профессионалам, к тому же она устарела, и в ней используются неочевидные утверждения из теории множеств. И да, уравнений там нет.

Алгебра, 10-11 классы - Мордкович, у него есть как учебник, так и задачник.

>>74392
вот в этом файле содержится большая часть школьной геометрии. Рекомендую прочитать первую главу из него вкупе с Атанасяном.
>> No.74476 Reply
>>74456
Как это нет? Открываем. Смотрим. Первые бросившиеся в глаза задания 4 и 5 решаются банальным представлением в виде геометрического ряда.
>> No.74477 Reply
>>74437
> сильнейший из ныне живущих аналитиков
Это кто?
>> No.74479 Reply
>>74476
Алсо, остальные "вычислить с погрешностью" делаются тоже разложением в ряд.
>> No.74481 Reply
>>74477
Доброчую.
>>74437
Доставь статью, интересно же.
>> No.74482 Reply
File: Hacker-s-Delight-(2002).pdf
Pdf, 4.97 KB, 271×424 - Click the image to get file
Hacker-s-Delight-(2002).pdf
>>74277
Вставлю свои пять копеек. Сильно не бить, я только на 4 курс перешёл, но интересующие меня предметы изучал глубоко и полно, поэтому часть заполню. По некоторым предметам пользовался электронными лекциями своего препода, поэтому освещать их смысла не вижу.

Алгебра - Винберг - Курс Алгебры. Рекомендую аккуратно на досуге почитывать ван дер Вардена, хотя он тяжелый и устаревший, но даёт, так сказать, широту охвата.

Дискретная математика - тут смотря что нужно. Я такого предмета не знаю, у нас были отдельно автоматы, та, булевозначный анализ, матлогика. По автоматам - Хопкрофт - теория автоматов, лучше английский, но русский из файн ту, по ТА - Роджерс - теория рекурсивных функций и эффективная вычислимость, но вряд ли кому-нибудь будут нужны такие ебеня теории вычислимости, по булану - Whitesitt - Boolean Algebra and its applications, по матлогике - Ершов Палютин - Математическая логика.
Алсо, у нас в НГУ есть книфка Косточка - Дискретная математика. Там основы в кратком изложении, не знаю как там всё написано, но пацаны с физфака нахваливали.

Дифференциальные уравнения - обычные : Понтрягин, урматы : Тихонов, Самарский. Много, сердито, сложно, ну а вы как хотели?

Дифференциальная геометрия - тут смотря что имеется ввиду. Риманова у нас идёт отдельно, в дифгеом включалась только магия навроде кривизны кривых, всякие опервые/вторые квадратичные формы поверхностей, совсем немного про гомотопию и всякие эйлеровы характеристики, немного про неевклидовы геометрии, немного про векторные поля на поверхностях.
Из книг рекомендую Погорелов - Дифференциальная геометрия - это про поверхности, читать просто и приятно глазу, Spivak - A comprehensive introduction to differential geometry - это немного более в сторону другого курса, но тоже сойдёт. Ещё почитывал классику в виде Kreyszig - Differential Geometry и Thurston - The geometry and topology of three-manifolds, но они скучны как по мне.
Если начать говорить про Риманову геометрию :
Хирш - Дифференциальная топология
Berger - A panoramic view of Riemannian geometry
Bishop - Tensor analysis on manifolds
Уорнер - Основы теории гладких многообразий и групп Ли
Из всего прочитанного (почти всё не до конца и вообще маловато) мне понравились именно эти книги. Все серьёзные, требующие хороший уровень подготовки (ну и вообще риманова геометрия читается только на 4 курсе, я то так, ради интереса поглядывал)

Топология - какая топология : общая, алгебраическая, дифференциальная? По последней я высказался выше, по алгебраической промолчу, ибо не изучалась она мной, по общей - Виро & co. - Элементарная топология и Engelking - General topology. Первая для начинающих, вторая - общий экскурс.

Функциональный анализ - Хелемский - Лекции по функциональному анализу и на русском только её и никакую другую. В колмогоровых фоминых, кутателадзах и иосидах ебнутый на всю голову и устаревший подход к топологии и банаховым пространствам. Если мозг не готов к категориям, то Эдвардс - Функциональный анализ, она более менее.

ТФКП - Markushevich - Theory of functions of a complex variable

Теория вероятностей, Математическая статистика - книги Боровкова по теории вероятностей и матстату. Больше ничего не нужно.

Методы оптимизации - а тут что конкретно? Если линпрог, то любую можно брать, не ошибётесь. Я к экзамену готовился по местной методичке и заодно Мину - Математическое программирование. Теория и алгоритмы

Теория чисел - Виноградов - Основы теории чисел, Ireland, Rosen - A classical introduction to modern number theory. Вторая более специальная, чем первая, уносит в сторону алгебраической тч, если для знакомства, то начинать лучше с первой.

От себя добавлю про теоркат, упоминавшийся здесь:
The Joy of cats, A Gentle Introduction to Category Theory, лень уже авторов вспоминать, сами найдёте. Эти две лучшие, на русском нормальной литературы нет и не предвидится. А про приложения - тут вопрос интересный, ибо теоркат сам по себе - это просто некий язык, на котором выражается вся математика. Есть приложения монад в программировании, в хачкелле, например. Для просветления читайте вики на хачкел.орг.
Напоследок прилепляю книжку, форсившуюся на нульче. Отличная книга для байтоёбов тех, кто интересуется многозначительно
>> No.74483 Reply
>>74479
> Первые бросившиеся в глаза задания 4 и 5
Ну это скорее разминка) Наверное, смысл в том, что дурак полезет дифференцировать дробь по формуле "отношение производных" и обломается.
> "вычислить с погрешностью" делаются тоже разложением в ряд
Не верю, что сможешь вычислить, например #13, разложив x^x в ряд. Там чуть хитрее, надо пооценивать подынтегральное выражение.
Я согласен, что "чистому" математику такое вряд ли пригодится, но вот в математике "второй культуры" встречаются и оценки и другие трюки.
>> No.74484 Reply
>>74483
в смысле, "производная отношения", самофикс
>> No.74486 Reply
>>74483
Но ведь для всего этого есть программные пакеты же.
мимоинженер
>> No.74490 Reply
>>74477
enwiki://Shing-Tung_Yau
>>74482
В треде появился человек, который сам читал то, что рекомендует.
И при этом рекомендует не фихтенгольца.
Ммда, это у нас редкость. Проходи, наливай себе чаю, бери конфетки.
> Винберг - Курс Алгебры. Рекомендую аккуратно на досуге почитывать ван дер Вардена
Всё так. Хотя многие добавили бы сюда Лэнга.
> Дифференциальные уравнения
А ты видел учебник Арнольда по ОДУ? Многие (и я в т.ч.) считают его лучшим учебником по ОДУ во всей Галактике!
> Дифференциальная геометрия
Ещё надо добавить книжечки J.M.Lee, они очень хороши.
> Виро & co. - Элементарная топология
Абсолютно точно. А после неё, кстати (или вместо), нужно обязательно почитать книжку Вербицкого по общей топологии.
> Функциональный анализ - Хелемский - Лекции по функциональному анализу и на русском только её и никакую другую. В колмогоровых фоминых, кутателадзах и иосидах ебнутый на всю голову и устаревший подход к топологии и банаховым пространствам.
Именно так.

А куда ты собираешься поступать после НГУ?
ну не в аспирантуру же НГУ, лол
>> No.74492 Reply
>>74490

Привет.
Это ты "человек, который в состоянии без подготовки понять большую часть статей из основных математических разделов Архива"?
Если нетрудно, можешь ответить на несколько вопросов?

1. посмотри список книг на studium: http://lj.rossia.org/community/studium/1717.html. Что бы ты добавил/убрал? По каким книгам и лекциям учишься сам?

2. Допустим, у меня похожая цель - разбираться в актуальных статьях с архива. Но в названных тобой разделах математики появляется по десятку статей в день. Ты их все читаешь? Что делаешь, если какую-то статью не получается понять?

3. Ты планируешь уехать/уже уехал? Есть ли у "чистых" математиков перспективы работы в России?
>> No.74493 Reply
>>74486

Программные пакеты помогают в конечных вычислениях, но не решают всех проблем. Всё равно нужно понимать, что именно делать и зачем. Например, легко представить ситуацию, когда для решения задачи нужно оценить асимптотику какого-нибудь интеграла или суммы.
>> No.74495 Reply
>>74492
> разбираться в актуальных статьях с архива
Зачем?
>> No.74500 Reply
>>74350
> вычисления с десятипроцентной точностью среднего от сотой степени синуса
Как вычислить это?
>> No.74508 Reply
>>74492
> "человек, который в состоянии без подготовки понять большую часть статей из основных математических разделов Архива"
Это определение математика. Я ещё только учусь :3
> 1. список книг
Этот список представляет собой совокупность рекомендаций Миши Вербицкого и Димы Павлова. Я согласен со всем, что там имеется (и сам раньше руководствовался этим списком), но там нет ничего по гомологической алгебре, мало по алгебраической геометрии и очень мало по теории чисел.
> 2. разбираться в актуальных статьях с архива.
Нет необходимости читать всё подряд. Все статьи просматривают только люди, которые непосредственно практикуют данную науку. Да и то, не разбирают статьи, а в большинстве случаев читают только абстракты. В учебных целях нужно подойти к старшему товарищу и попросить его составить список ключевых, штук 10, статей по какой-то конкретной науке за последние 10 лет.
> 3. Есть ли у "чистых" математиков перспективы работы в России?
Почти нет. Преподавать в МГУ берут своих недоучек, лояльных начальству, а на Матфаке почти все молодые преподы -- выпускники НМУ с западным phd (т.е. тоже "свои").
А по какому принципу предлагают позиции в Стекловке науке до сих пор неизвестно, лол.
>> No.74509 Reply
>>74500
> среднего от сотой степени синуса
А что это значит?
>> No.74514 Reply
File: 1267899032467.jpg
Jpg, 11.29 KB, 313×371 - Click the image to expand
edit Find source with google Find source with iqdb
1267899032467.jpg
>>74490
> А ты видел учебник Арнольда по ОДУ?
Нет, к сожалению. Я читал у него книжку по механике (классические методы современной механики, как-то так, лень гуглить). Великолепная книжка.
> Ещё надо добавить книжечки J.M.Lee, они очень хороши.
Не читал, возьму на заметку.
> , нужно обязательно почитать книжку Вербицкого по общей топологии.
А у него есть по общей топологии? Не знал даже.
> А куда ты собираешься поступать после НГУ?
За бугор. Уже есть тоефл, надо гре и всё. Если не получится, то в магистратуру НГУ и повторять процесс.
>>74492
Я не он, но можно попробую ответить?
> Что бы ты добавил/убрал? По каким книгам и лекциям учишься сам?
Убрал бы кучи книг из раздела Алгебра и Топология, в Set theory добавил бы книжку Архангельского, добавил бы ко всем разделам что я рекомендовал выше, убрал бы полностью теорию чисел, разделил бы анализ на Calculus и Analysis, а топологию на общую, алг. и дифф. с использованием моей литературы, что такое complex geometry я вообще не очень представляю, скорее всего речь о дифгеоме. В общем, поправить есть что.
> 2. Допустим, у меня похожая цель - разбираться в актуальных статьях с архива
А смысл?
> Но в названных тобой разделах математики появляется по десятку статей в день
Дело в том, что математик (эта каторый учоный ёба) специализируется в очень конкретной области. Это нормально, ибо теоретики сегодня на триста вёрст обогнали практиков, поэтому широкий охват нынче невозможен и так или иначе нужно сужать область исследования до раздела в разделе в разделе алгебраической геометрии, например, а смежные полистывать на досуге. Меня сейчас ссаными тряпками закидают, но лучше делать так, это в университете всё валят скопом, в науке надо конкретно и по делу. В голове даже из той же алг. геометрии всё не удержишь.
> Есть ли у "чистых" математиков перспективы работы в России?
Смотря каких. Чистых совсем мало сейчас, только что топологи, матанщики (это я так всех, кто analysis занимается называю) и алгебраисты. У нас первые и третьи живут ништяк, особенно топологи, вечно по конференциям забугорным, да все в новых штанах да с айфонами ходят.
>> No.74515 Reply
>>74514
> Убрал бы кучи книг
Убирать там ничего не надо, список заведомо составлен так, чтобы книжки друг друга частично перекрывали.
Кроме того, как раз в разделах Алгебра и Топология нет и половины того,что нужно изучить за бакалавриат по соответствующим дисциплинам.
Другое дело, что заголовки разделов не совсем удачно расставлены.
> что такое complex geometry
Это геометрия над комплексными числами :3. Самый быстроразвивающийся раздел геометрии на текущий момент.
Она не алгебраическая и не дифференциальная, а смесь: в алгеме уже давно практикуются аналитические методы и наоборот. Деление на алгебраическое/дифференциальное во всей геометрии и топологии уже скорее дань традиции.
> до раздела в разделе в разделе
Определение, которое я давал, требует не умения понимать статьи в одном разделе, а во всех основных разделах (ну, хотя бы в четырёх).
Как раз мудаков, которые всю жизнь занимаются одной узкой темой и ничего больше не знают -- пруд пруди. Они засоряют пространство своими бездарными статьями, которые никому не нужны, кроме нескольких штук таких же "узких специалистов" и ведут к разделению, а не объединению математической науки. Они, конечно, заслуживают профессиональную дисквалификацию, если бы такая существовала, лол.

(>>74508 и >>74490 это один человек)
>> No.74519 Reply
>>74515
> Как раз мудаков, которые всю жизнь занимаются одной узкой темой и ничего больше не знают -- пруд пруди.
Мудаков, которые разбираются во многих основных разделах, но ни в одном не являются разработчиком, еще больше.
>> No.74522 Reply
>>74444
Какая ерунда. Солидарен с >>74447-куном.
>> No.74523 Reply
>>74459
А ты знаешь, что такое натуральные числа? Структуру ы студию.
>> No.74524 Reply
>>74519
Между прочим, годная позиция. Интересно, в этом треде есть "разработчики"?
>> No.74525 Reply
>>74514
> > 2. Допустим, у меня похожая цель - разбираться в актуальных статьях с архива
> А смысл?
Ну это гипотетически, если я буду серьёзно заниматься наукой.
>> No.74527 Reply
>>74519
Разбираться во многих разделах, очевидно, сложнее, чем разбираться в одном, поэтому таких людей, конечно же, значительно меньше, чем "узкоспециализированных" мудаков.
> разработчиком
Вот здесь корень зла.
Математик это не тот, что решает бессмысленные задачи и пачкает своими "результатами" бумагу.
Наука существует до тех пор, пока есть люди, которые её понимают.
Смысл существования математика исключительно в одном: это хранитель науки.
Задача математика -- понимать как можно больше и, самое главное, понимать взаимосвязи. А уже на основе этого понимания можно принимать участие в развитии математической теории.
>> No.74529 Reply
File: Кенни.jpg
Jpg, 26.25 KB, 549×537 - Click the image to expand
edit Find source with google Find source with iqdb
Кенни.jpg
>>74500
Ну ответьте же кто-нибудь.
>> No.74531 Reply
>>74492
Во-первых, это, кажись, не он. Во-вторых, я тоже не он.
По (1) скажу, что убрал бы почти всё.
Начальный уровень:
Феферман Соломон, "Числовые системы", начать с неформальной аксиоматической теории множеств;
Ленг Серж, "Алгебра" (был там в категории "Алгебра".);
Ван дер Варден, "Алгебра", хотя это низкий уровень.
Mathematical logic:
Линдон Р., "Заметки по логике";
Смальян Р., "Теория формальных систем";
Шенфилд Дж., "Математическая логика".
Set theory:
Коэн Пол, "Теория множеств и континуум-гипотеза";
Ван Хао, Мак-Нотон "Аксиоматические системы теории множеств".
Дальше Бурбаки.
В логику ещ нужно добавить что-то о конструктивизме и интуиционизме, эти ветви тоже изучить (тут я пока не разбираюсь).
>> No.74536 Reply
>>74527
> Математик это не тот, что решает бессмысленные задачи и пачкает своими "результатами" бумагу.
В основном математическая теория развивается под действием потребности решения возникающих задач.
> Задача математика -- понимать как можно больше и, самое главное, понимать взаимосвязи. А уже на основе этого понимания можно принимать участие в развитии математической теории.
А что будет толчком к развитию? На одном понимании можно разве что работать только над систематизацией старого.
>>74529
Здесь одни хранители науки, еба.
>> No.74538 Reply
>>74529
Непонятно, какое именно среднее. Если матожидание, то что за функция распределения?
Скорее всего Арнольд имеет в виду не конкретный пример, а то, какие интегралы нужно уметь быстро прикинуть на бумажке.
А конкретные примеры есть в тривиуме.
>> No.74539 Reply
>>74536
Математика развивается, когда кто-то руководствуясь исключительно своими пониманием и интересом выдвигает гипотезу и либо сам на её основании строит теорию, либо представляет свою гипотезу математической общественности. "Решатели задач" -- люди второго класса.
>> No.74540 Reply
>>74536
Кстати, я уже запутался.
Ты тот из НГУ или кто-то из местной шелупони?
>> No.74542 Reply
>>74539
Часто новые теории строятся как вспомогательные для решения задач.
>> No.74545 Reply
>>74539
> "Решатели задач" -- люди второго класса.
>>74540
> или кто-то из местной шелупони?
Мат-илита в треде?
>> No.74546 Reply
>>74542
Есть фундаментальное противоречие между "theory builders" и "problem solvers", речь об этом.

В общем, надоело мне метать бисер, скрою ка я тред.
>> No.74548 Reply
>>74539
> выдвигает гипотезу
Такое случается в основном в теории чисел. Ты только с этой областью знаком?
К слову о ТЧ, человек второго сорта, Виноградов, решая проблему Гольдбаха, создал метод тригонометрических сумм.
>> No.74549 Reply
>>74548
о том и речь, пока одни строят алгебраическую геометрию, другие занимаются тригонометрическими суммами, лол
>> No.74550 Reply
>>74549
> пока одни строят алгебраическую геометрию
Будто что-то хорошее.
>> No.74559 Reply
>>74443
Удваиваю, этого охуенного математика давно пора заигнорить. Ему бы очень пригодился принудительный трипкод, ящитаю. Текста много, начинаешь читать - а ни одного ответа на вопрос нет. Да ну нахуй.
>> No.74563 Reply
File: measure-theory.jpg
Jpg, 52.65 KB, 600×375 - Click the image to expand
edit Find source with google Find source with iqdb
measure-theory.jpg
>>74559
Утраиваю. Пускай оставит нас, "местную шелупонь", в покое. Здесь ему не рады.
>> No.74569 Reply
File: b50759153b13.jpg
Jpg, 28.50 KB, 604×281 - Click the image to expand
edit Find source with google Find source with iqdb
b50759153b13.jpg
>>74559
>>74563
Да ладно вам, ворчуны но мне кажется, что это семён.
Я же сюда поразвлекаться заходил. То есть все мои высказывания остаются в силе, но я заранее знал, что уровень местной аудитории очень низок и мало кто вообще сможет понять, что я имею ввиду. Рад, что моё высокомерие вызвало такой ягодичный зуд, lol.
Зато несколько человек, которые действительно хотели вынесли из этого треда что-то полезное, получили от меня советы мудрые по литературе.
>> No.74570 Reply
>>74569
унизили@назовусь_троллем.жпг
>> No.74572 Reply
>>74523
> А ты знаешь, что такое натуральные числа?
Множество с аксиомами Пеано.
>> No.74573 Reply
File: Чекист.jpg
Jpg, 34.42 KB, 250×251 - Click the image to expand
edit Find source with google Find source with iqdb
Чекист.jpg
>>74563
А, ладно, так и быть. Учетверяю.
>> No.74576 Reply
>>74573
Ну вот, если кто-то упятерит, то мне по неписанным правилам действительно придётся покинуть тред!
Но где я тогда буду демонстрировать высокомерие?
Пожалуйста, не упятеряйте!
====
Кстати, куда пропал анон из Казахстана, который желал дистанционно науку постигать?
>> No.74579 Reply
>>74569
Тебе про другое весь тред твердят, няша. Хотя вот это уже голос со стороны...
> уровень местной аудитории очень низок и мало кто вообще сможет понять, что я имею ввиду.
Во-первых, нет. Во-вторых,
> Текста много, начинаешь читать - а ни одного ответа на вопрос нет. Да ну нахуй
Тебе всё уже сказали.
> но мне кажется, что это семён
Ага, нестерпимая анальная боль вызывает галлюцианации... Хотя стоп, это же не двач. Выпей чаю и перепрячь зефирки. Добра!
финансист-кун
>> No.74581 Reply
>>74579
> > Текста много, начинаешь читать - а ни одного ответа на вопрос нет. Да ну нахуй
Так меня никто ни о чём не спрашивал, lol. А те, кто спрашивал, советов мудрых получили, никого не обделил.
> Во-первых, нет
Много ли здесь людей, который хоть раз в жизни сталкивались с понятием схемы, производной категории, изучали теорию препятствий? (это не "сливки науки" а самая-самая база)
>> No.74582 Reply
> Так меня никто ни о чём не спрашивал
Ни о чём конкретном, я имел ввиду.
>> No.74585 Reply
File: grothendieck_1.jpg
Jpg, 10.90 KB, 268×326 - Click the image to expand
edit Find source with google Find source with iqdb
grothendieck_1.jpg
>>74515
> Убирать там ничего не надо, список заведомо составлен так, чтобы книжки друг друга частично перекрывали.
Ну тогда уж конкретизируйте, а то когда я вижу, например, Алгебру - я сразу представляю этакий базовый курс вышки безо всяких там углублений в теорию колец, групп или прости Хоссподи, характеров. Соответственно, книжки Винберга хватит вполне.
> Это геометрия над комплексными числами
Такого понятия я не знаю. Если имеется ввиду просто комплексные многообразия и ТФКПшный агрегат сверху, тогда понятно (не совсем правда понятно на кой это отдельно выделять, бонусов мне с моей колокольни не видно).
> Определение, которое я давал, требует не умения понимать статьи в одном разделе, а во всех основных разделах
> Как раз мудаков, которые всю жизнь занимаются одной узкой темой и ничего больше не знают -- пруд пруди.
> Они, конечно, заслуживают профессиональную дисквалификацию
Ты не понял ничего. Повторяю ещё раз : профучоный на данный момент не в состоянии охватывать много разделов сразу. Он может владеть основами смежных тем, я это категорически приветствую, но копать он будет вынужден исключительно в своей норке, иначе он будет знать по чуть-чуть отовсюду и ничего не откроет, а если нет результатов, то ученые быстро закисают и валяют дурака. Есть разница между изучением смежной области в процессе открытия чего-то в своей конкретной и изучения этой смежной области потому что это ниибаца не по мудацки.
> Они засоряют пространство своими бездарными статьями, которые никому не нужны, кроме нескольких штук таких же "узких специалистов"
Не всем быть Перельманами. Лучше открывать что-то своё в своей области, причём, как правило, если есть результат, то он как-то обобщается и становится уже более весомой штукой. Ну или не обобщается, далее зависит от специализированности открытия.
Вообще, это вопрос очень дискуссионный, обсасывался не раз и не два, аргументы обеих сторон все знают, так что можно сворачивать этот балаган.
>>74527
> Смысл существования математика исключительно в одном: это хранитель науки.
Смысл математика - это делать теоретические открытия не взирая на практику, а просто ради дальнейшего прогресса математического познания. Например, вся эта бинарная арифметика, которая заложена в эти ваши современные кампьютиры придумана ещё в 19 веке и вовсе не для этого.
>>74536
> В основном математическая теория развивается под действием потребности решения возникающих задач.
Это прошлый век. Теория улетелела в такие атсральные дали, что практика не догонит никогда. Практика активно сейчас пожирает теорию 50 летней давности, как например топологию или теоркат тот же.
> А что будет толчком к развитию?
Толчком к развитию, как правило, будет то, что ты помогаешь науке, человечеству и прогрессу. Это такая метафизика, математики недаром ведь почти все хорошие философы были.
> На одном понимании можно разве что работать только над систематизацией старого.
Хорошо систематизировать тоже надо уметь.
>>74539
> Математика развивается, когда кто-то руководствуясь исключительно своими пониманием и интересом выдвигает гипотезу и либо сам на её основании строит теорию, либо представляет свою гипотезу математической общественности
Почти так, да. Соглашусь. Штука в том, что процесс конструктивен и итерационен, после первого бага в теории появляется перепиленный вариант и тд до победного конца или фейла. Пока одни строят, другие ломают головы в процессе одобрения или отвергания. Попутно рождая, кстати, другие теории и забивая в итоге на первоначальные.
>>74540
Это НГУ-кун. Мои посты http://dobrochan.org/u/res/74216.xhtml#i74514 и http://dobrochan.org/u/res/74216.xhtml#i74482
>>74548
> Такое случается в основном в теории чисел
Внезапно это ещё почему?
>> No.74587 Reply
File: picture.jpg
Jpg, 23.74 KB, 488×356 - Click the image to expand
edit Find source with google Find source with iqdb
picture.jpg
Есть здесь школьные преподаватели? А то скоро предстоит проходить практику, хотелось бы уточнить вопросы по ней и по дальнейшей работе.
1) в какие школы по глубине изучения математики и классы обычно присылают практикантов? как эти школы выбираются?
2) как проверяют мою профпригодность?
3) я так понял, по началу урок будет вестись вместе со штатным преподом?
4) ставят сразу на алгебру и геометрию, или могут поставить преподавать что-то одно?
5) дают ли программы и обязан ли им четко следовать? как происходит контроль?
6) домашка тоже как-то регламентирована или я могу давать задачи на свое усмотрение?
7) вообще какие подводные камни?

Куда еще могут направить учащегося математика (факультет не педагогический, но педагогику изучаем) на практику?
>> No.74588 Reply
>>74579
>>74314

Привет. Так ты сделал study group по финансам? Я бы поучаствовал.
У меня 1-е математическое образование и год экономической магистратуры.
>> No.74589 Reply
>>74306
Ты охуенен. Я бы, наверное, не стал бы читать эту книгу, если бы ты её так не назвал. Я не знаю почему, ведь я бы сам мог её так назвать... но мне теперь так легко и приятно её читать! Это волшебно.
>> No.74592 Reply
>>74585
> Это НГУ-кун
Какой курс?
>> No.74593 Reply
>>74585
> базовый курс вышки
Трудно сказать, что такое "базовый курс алгебры". Я как-то посещал базовый курс, в котором были представления конечных групп, теория Галуа и алгебраические многообразия, lol
> на данный момент не в состоянии охватывать много разделов сразу
Разумеется, но должен в первую очередь к этому стремиться. Иначе -- не учёный, а реакционер и вредитель.
> Лучше открывать что-то своё в своей области
> Вообще, это вопрос очень дискуссионный
Да, это всё долгие разговоры. Суть в противостоянии мировоззрений: Строители теорий против Решателей задач (я, конечно, за строителей)
> Смысл математика - это делать теоретические открытия не взирая на практику, а просто ради дальнейшего прогресса математического познания.
Именно так. Но задача №0 -- сохранить то, что имеется.
> Это НГУ-кун. Мои посты
Ну вот, а я побеседовал с каким-то хуем. Мне кажется, в этом треде пишут всего человек четыре-пять, причём пара мудаков со мной пытается спорить и мне приходится разжёвывать им одно и то же по нескольку раз.

>>74589
Рад, что тебе понравилось :3
>> No.74594 Reply
File: 0_22eb0_4ea352fc_L.jpg
Jpg, 33.98 KB, 425×282 - Click the image to expand
edit Find source with google Find source with iqdb
0_22eb0_4ea352fc_L.jpg
Итак, вносим коррективы:
Математический анализ - Фихтенгольц. Курс дифференциального и интегрального исчисления (или же Зорич, не принципиально)
Алгебра - Винберг. Курс Алгебры.
Аналитическая геометрия - ?
Линейная алгебра и геометрия - ?
Дискретная математика - ?
Математическая логика - Ершов, Палютин. Математическая логика
Дифференциальные уравнения - Понтрягин. Обыкновенные дифференциальные уравнения
Дифференциальная геометрия - Погорелов. Дифференциальная геометрия
Топология - Виро. Элементарная топология
Функциональный анализ и интегральные уравнения - Хелемский. Лекции по функциональному анализу
Теория функций комплексного переменного - ?
Уравнения в частных производных (вместо этого курса физикам читаются Методы математической физики) - ?
Теория вероятностей - книги автора Боровкова
Математическая статистика - книги автора Боровкова
Теория случайных процессов - ?
Вариационное исчисление и методы оптимизации - ?
Методы вычислений, то есть численные методы - ?
Теория чисел - Виноградов. Основы теории чисел

==============================================
>>74576
> Кстати, куда пропал анон из Казахстана, который желал дистанционно науку постигать?
Я здеся. Вот, решил проявить инициативу и вместе с вами составить список книг для самообразования.
>> No.74595 Reply
>>74594
дам дружеский совет: не изобретай велосипед, пользуйся списком Вербицкого, он заведомо компетентнее и умнее нас всех вместе взятых
>> No.74596 Reply
File: iom2i1.jpg
Jpg, 24.78 KB, 449×500 - Click the image to expand
edit Find source with google Find source with iqdb
iom2i1.jpg
>>74595
> > он заведомо компетентнее и умнее нас всех вместе взятых
>> No.74598 Reply
>>74595
Дай ссылку.
>>74596
Врёт?

Я, тащемта не разбираюсь в этих ваших вербицких, просто хочу увидеть уже конкретный и по возможности минимальный список того, что я мог бы начать читать прямо сегодня. В конце-концов, когда я прочитал бы "основы" я бы в некоторой степени и сам смог бы оценить, что требует внимания больше, а что - меньше.
>> No.74599 Reply
>>74598
А каковы твои цели?
>> No.74600 Reply
>>74599
Бля. У меня много целей. Ближайшие - съесть тортик, посмотреть Mirai Nikki и пойти спать.

А каковы цели у студента, поступающего в ВУЗ? Я не дожидаясь ответа скажу, что чаще всего - никаких, кроме пары заблуждений. И, что характерно, многие из этих "бесцельных", которые делают какую-то хуйню просто из интереса, не только не отваливаются на ближайшей сессии, но и, зачастую, чего-то добиваются.
Вот только некоторые предпочитают не пользоваться услугами гос.учреждений.
>> No.74602 Reply
>>74600
Куда поступил? Какая специальность?
>> No.74603 Reply
>>74602
> Вот только некоторые предпочитают не пользоваться услугами гос.учреждений.
Хватит задавать вопросы уже. Дайте кто-нибудь ответ, наконец. Мне похуй. если он будет неверным, я буду рад, если он просто будет конкретным.
>> No.74604 Reply
File: d40g2ycxl3yb[1].jpg
Jpg, 39.15 KB, 594×666 - Click the image to expand
edit Find source with google Find source with iqdb
d40g2ycxl3yb[1].jpg
> Вот только некоторые предпочитают не пользоваться услугами гос.учреждений
> не пользоваться услугами гос.учреждений
> не пользоваться
> Куда поступил? Какая специальность?
И вот с людьми подобного сорта я беседую о сущности математики.
>> No.74605 Reply
>>74600

%%> Mirai Nikki
Мне не понравился, дропнул%%

Выше давали ссылку: http://lj.rossia.org/community/studium/1717.html
Там книги идут по возрастанию сложности. Если плохо с английским, обязательно прокачивай.
>> No.74606 Reply
>> No.74607 Reply
>>74605
Спасибо. Вот только непонятен порядок прочтения. Мой английский вполне неплох.
а дневничок нравится. Юно - няшка. Да, у меня своеобразные предпочтения.
>>74606
Вот, спасибо, отлично. Вербицкий, так Вербицкий. Как минимум, не придётся выбирать между двумя стогами сена.
>> No.74608 Reply
>>74607
Вообще-то мой совет был забить. Ссылку я кинул по просьбе.
>> No.74609 Reply
>>74608
Но я не просил совета, а ссылку ты кинул. Спасибо.
>> No.74615 Reply
>>74587
Бамп вопросу.
>> No.74621 Reply
>>74572
Прекрасно. Также ты мог говорить не об интерпретации натуральных чисел в теории множеств, а о чистой теории натуральных чисел.
>> No.74624 Reply
>>74572
и что характерно, он снова не ответил на вопрос. заигнорь уже его.
>> No.74627 Reply
>>74588
People united is a wonderful thing.

Ты вот первый откликнувшийся. Можем попробовать осилить. Собственно, финансы хороши тем, что есть биржа, на которой можно ставить эксперименты. С другой стороны, можно делать научную работу, если ты числишься в соответсвующем институте. Я вот не числюсь, и это больше как хобби. Поэтому я предлагаю программу, по которой готовят "кванта".

Сначала приглашаем всех желающих читать "опционы" Халла. Эта книга есть необходимый минимум, чтобы тебя взяли на работу в банк. Поэтому заинтересованные лица появятся. Читаем. Там 40 глав, в серединке хардкорные, но я смогу объяснить математиику не продвинутым анонам, а ты уж точно поймешь. Это фундамент. А дальше надо запилить несколько отдельных совместных чтений по:
Introductory analysis - у кого совсем плохо с матаном
Complex analysis - ТФКП
Probability Theory and Stochastic Processes - статы
Basic mathematical finance - формула Блэка-Шолза
Medium mathematical finance - скорей всего, начальные финансы - это классический подход без мартингалов, а потом все то же самое, только с "хорошой мерой".
Interest rate modelling
Credit Derivatives
Numerical Techniques
Две последние для того, чтобы запрогать большую часть моделей.
C++
C++ and quant finance

Как-то так. Добра!
>> No.74628 Reply
>>74216
Анон, я хочу понять теоремы Гёделя с доказательствами. Многое ли необходимо знать для этого? Что нужно прочитать?
>> No.74629 Reply
>>74627
Я, пожалуй, присоединюсь на досуге. Заодно хачкель попрактикую, он у них популярен.
>> No.74632 Reply
>>74628
Писал выше: >>74531.
Алсо, на мой взгляд это не математика а узкая область философии.
>> No.74633 Reply
>>74630
Сам ты узкая область философии. Нет тебе места на Земле.
>>74628
Нет, не много. Они очень простые, на самом деле, в своих доказательствах. Другое дело, что выводы ебаутые получаются, но следить за доказательством - очень просто, не нужно знать фактически ничего. Все книжки по логике, которые я видел, подавали это в сноном виде. Алсо, есть науч.поп. книжка: "Гёдель, Эшер, Бах, эта бесконечная гирлянда". Много слов, ну и вообще, науч.поп. же, но очень просто и понятно.
>> No.74635 Reply
>>74633
> Сам ты узкая область философии. Нет тебе места на Земле.
Не понял, объясни. Особенно жду по первому тезису.
>> No.74648 Reply
>>74604
На самом деле ошибку допустил тот, кто считает "взятие интегралов=нахождение первообразных элементарных функций бла-бла". Я даже комментировать не хочу, почему это неверно, неточно и вообще в действительности обнажает категоричность и низкий уровень отдельно взятого критикана.
Это и есть мат. сноб, вещавший о низком уровне, бисере и который всегда что-нибудь ляпнет, да в 50% случаев неверно
мимосозерцал
>> No.74671 Reply
>>74531
> Дальше Бурбаки.
Не нужны.
>> No.74672 Reply
>>74594
> Дискретная математика
Новиков. Дискретная математика.
> Математический анализ - Фихтенгольц. Курс дифференциального и интегрального исчисления (или же Зорич, не принципиально)
Не Фихтенгольц, а Гурса.
> Алгебра - Винберг. Курс Алгебры.
Ван дер Варден же. http://mburyakov.ru/phtf/lib/Varden.pdf
>> No.74675 Reply
>>74627
Как используется ТФКП в финансовой деятельности?
Есть раздел computer science, который тесно связан с работой финансиста?
>> No.74689 Reply
>>74675
> Как используется ТФКП в финансовой деятельности?
Во-первых, там не передовой ТФКП. Вузовской программы должно хватить. Во-вторых, интегрирование по контуру и преобразование Фурье - это неотъемлимые умения работающего кванта.
> Есть раздел computer science, который тесно связан с работой финансиста?
Омг. Ты, наверное, ничего не слышал про это.Там дохуя различных разделов. Самый яркий пример - это hft. Но я не понимаю ничего в разделах, которые там используются (какие-то нестационарные ряды...). А для алгоритмических торгов применяются теории динамических систем, теории оптимизации, распределенные вычисления (Concurrent, parallel and distributed systems - вроде бы это самая актуальная область их применения). В базовый набор курсов сюда еще входят теория графов и комбинаторная геометрия.

Алсо если ты спрашиваешь, как кодер, который пытается маскироваться под высокую науку, то спешу тебя обрадовать: здесь, чтобы закодить матан, надо действительно его понимать. То есть, например, простейшее моделирование interest rate, да даже цен на опцион подразумевает следующее.

У тебя есть базовый актив, цена акции, она двигается со временем. В дискретном представлении это граф. Грубо говоря из цены 1.47 она может двигаться в 1.46 и 1.48, из 1.46 в 1.45 или 1.47. На этой сетке есть своя вероятность, нам неизвестная, и, самое интересное, ненужная. На этом же графе для каждой цены акции есть цена производного инструмента на акцию. В случае европейского опциона цена есть просто мат.ожидание выплат по специальной мартингальной мере. Понятно, что в случае, когда r - interest rate (так сказать, процент с гос.облигаций) меняется, а сам дериватив есть не опцион, а какой-нибудь CDS, задача усложняется. Иногда возможно привести аналитическую формулу, но ее все равно надо кодить, а потом условия на рынке изменятся и надо будет заново считать. Поэтому надо строить более тонкую модель. Алсо, конечно, всегда можно запилить Монте-Карло, но обычно грубый способ в лоб применяется только, чтобы проверить (хотя бы качественно) то, что ты написал.

Черт, чтобы объяснить лучше, надо взять конкретный пример, но в нем ничего не понятно без подготовки. А так это выглядит как набор безумных слов. Надеюсь, что хоть как-то ответил на вопрос.
>> No.74693 Reply
>>74672
Вообще-то Новиков - фигня, но мне вот про Ван Дер Вардена. В нём нет упражнений, насколько я вижу. Откуда выполнять упражнения?
>> No.74695 Reply
>>74648
да-да-да, я уже понял свои ошибки
Беседы можно вести только с теми, кто этого заслужил.
>> No.74696 Reply
>>74689
> если ты спрашиваешь, как кодер
Нет, я математик. Ну, учусь.

А ты уже работаешь?
>> No.74699 Reply
>>74696
Уже не работаю, лол. Алсо, по жизни занимаюсь совсем другими вещами. Так что мой интерес чисто любительский. Но да, для полноценной финансовой кафедры нужен кто-то с опытом senor position.

С другой стороны, не думаю, что в /u/ бывает как-то иначе. Например, в этом треде все любители. И любителя обсуждают программу самостоятельных занятий по математике - это по мне. Ровно за этим и существует этот раздел, разве не так?
>> No.74729 Reply
>>74689
> комбинаторная геометрия
Что это? Чем она занимается? В википедии есть такая категория, а статьи с определением не вижу. Есть какая-нибудь книга "комбинаторная геометрия для чайников"?
>> No.74731 Reply
>>74729
Википедия несовершенна.
http://dic.academic.ru/dic.nsf/enc_mathematics/2294/
>> No.74737 Reply
Матемачан, у меня бида. Внезапно понял, что совсем забыл математику и геометрию. Ну, то есть таблицу умножения ещё помню, и равнобедренный треугольник от равностороннего отличить в состоянии, но всё, что сложнее, отшибло напрочь. Посоветуй каких-нибудь хороших учебников, пожалуйста.
>> No.74757 Reply
>> No.74768 Reply
>>74737
> Посоветуй каких-нибудь хороших учебников, пожалуйста.
Ты бы хоть тред-то почитал. Мы как раз и разбираемся, что такое "хороший учебник" и составляем их список.
>> No.74771 Reply
>>74768
Так ему ведь совем для лохов надо, что ты пиздишь. Очень простой и ясный вопрос, только я не знаю ни одной книжки для такого лвла. В самом деле и сам бы был не прочь повторить школьную программу в сжатом виде, ну, может чуть продвинутую.
>> No.74774 Reply
>>74771
"Мордкович А.Г. - Алгебра и начала анализа 10-11 кл. - 2001"
http://rutracker.org/forum/viewtopic.php?t=2852759

"Учебник по геометрии 7-9 класс"
http://rutracker.org/forum/viewtopic.php?t=1299484

"Учебник по геометрии 10-11 класс"
http://rutracker.org/forum/viewtopic.php?t=1888459
>> No.74775 Reply
>>74350
Это типо как трёхзначные числа в третью степень возводить на время?
>> No.74776 Reply
>>74775
Это типо умение решать примерчики уровня 2+2.
>> No.74778 Reply
>> No.74787 Reply
Наверное, сюда.
Аноны, кто-нибудь пробовал представить в уме n-мерное пространство? Не цифрами, векторами и прочими математическими знаками, а именно увидеть его? Или это невозможно?
Сейчас представляю в уме трёхмерный куб, но как только хочу втыкнуть куда-нибудь ещё одну плоскость — у меня фейл.
>> No.74789 Reply
>>74787
Пробовал, и даже почти получается. http://anydigits.blogspot.com/2008/02/blog-post_06.html
>> No.74790 Reply
>>74787
Хотя это принципиально возможно, я думаю что это трудноосуществимо. Мне кажется некоторый шанс видеть в многомерном пространстве есть только у клиентов психиатрических больниц. Имея нормальную связь с реальностью этого не достичь. Для примера, попробуй посмотреть из двухмерного пространства. Спешу заверить, это не так просто. Потому что речь идет о не о том, чтобы представить себе плоскость в пространстве (трехмерном), а о том, чтобы представить себе плоскость БЕЗ пространства.
>> No.74791 Reply
>>74790
Что там двумерное - ты из нульмерного попробуй посмотреть, няша.
>> No.74792 Reply
>>74787
Вообрази прямую, которая имеет с трёхмерным пространством только одну общую точку. Потом вообрази четыре взаимоперпендикулярных вектора. А потом представь движение куба в трёхмерном пространстве, которое расположено под углом к нашему трёхмерному пространству.
>> No.74793 Reply
>>74792
Инженер три часа просидел на лекции математика, посвященной многомерным пространствам. В конце он, очень огорченный, подошел к лектору и сказал:
- Извините, я хотел бы хоть немножко представить себе предмет вашей лекции. Но я не могу вообразить сферу в девятимерном пространстве!
- Это же очень просто, - ответил ему математик, - вообразите сферу в N-мерном пространстве, а затем положите N равным девяти.
>> No.74864 Reply
>>74693
> В нём нет упражнений, насколько я вижу. Откуда выполнять упражнения
Кострикин - Сборник задач по алгебре. Учебник для вузов
>> No.74911 Reply
>>74778
Отрезок не дан же.
>> No.74916 Reply
>>74911
Синус как бе периодическая функция. Ясно, какой отрезок имеется в виду.
>> No.74926 Reply
>>74916
Мы не телепаты. Хрен его знает, какой он имел отрезок в виду.
>> No.74931 Reply
>>74926
> Мы не телепаты
Угу-угу. О том и речь.
>> No.74940 Reply
>>74916
Ну, допустим, имелся в виду [0,pi/2], тогда среднее равно 2/pi*int_{0}{pi/2}(sin(x))^100dx=99!!/100!!
Тривиум такой тривиум.
>> No.74942 Reply
>>74216
Аноны, чем дифференциал отличается от производной?
>> No.74945 Reply
>>74942
Производная это в некотором смысле коэффициент, число, свойство функции в точке. Дифференциал это приращение функции пропорциональное приращению аргумента и как раз тому самому коэффициенту. Если корче, то производная это k=dy/dx, а дифференциал это dy=k*dx.
>> No.74961 Reply
>>74942
Дифференциал это специальным образом определённое отображение касательных пространств.
Производной (точнее, производным отображением) обычно называют отображение, которое каждой точке сопоставляет дифференциал в этой точке.

Линейное отображение одномерных пространств задаётся одним числом (если говорить строго, "элементом поля"), поэтому в одномерном случае дифференциал является числом (по сути это тангенс угла наклона касательной в соответствующей точке), а производная -- функция, которая каждой точке сопоставляет дифференциал, т.е. этот угол наклона.
>> No.74962 Reply
>>74961
> т.е. этот угол наклона
этот тангенс угла наклона
fixed
Поэтому производную можно назвать "скоростью роста функции"
>> No.74963 Reply
>>74926
В периодическом случае можно просто говорить о среднем по всей прямой, вот и всё.
>> No.74968 Reply
>>74963
Ответ такой же, как и в этом случае >>74940
>> No.74969 Reply
>>74968
Не спорю.
>>74940
А дальше ты предлагаешь сделать сотню умножений и поделить? За пять минут?
Это ведь не доведено до ответа же.
>> No.74971 Reply
>>74969
Дальше можно преобразовать, как (2k-1)!!/(2k)!!=(2k-1)!/(2^k*k!)^2, и поиграться с формулой Стирлинга.
>> No.74975 Reply
File: Erdös.jpg
Jpg, 10.72 KB, 268×326 - Click the image to expand
edit Find source with google Find source with iqdb
Erdös.jpg
Посоветуйте стимуляторов для умственной деятельности.
>> No.74978 Reply
>>74975
тоже хочешь написать пару тысяч статей по комбинаторике?
>> No.74979 Reply
>>74971
С какого места формула Стирлинга даёт требуемую относительную погрешность?
Даже если забить на опечатку в формуле.
>> No.74980 Reply
Хочу написать хотя бы одну статью по математике, которая будет опубликована и одобрена каким-нибудь авторитетным батей. После чего смогу спокойно заняться другим хобби. Математического образования не имею. К самообразованию готов и полон энтузиазма.
Что делать?
Опережая вопрос "нахуя": чтоб было. Нет, я серьёзно.
>> No.74988 Reply
File: петросян845.jpg
Jpg, 58.58 KB, 443×600 - Click the image to expand
edit Find source with google Find source with iqdb
петросян845.jpg
>>74980
Ты бы еще спросил как нобелевку для лулзов получить.
>> No.74991 Reply
>>74988
Естественно, я жду ответа только от тех, у кого есть публикации, а не от тебя.
>> No.74992 Reply
>>74975
Пиздюль?
>> No.74993 Reply
>>74980
У тебя 3 пути:
1)Начать учить математику и лет через 5, при должном усердии и способностях ты сможешь написать статью на которую обратит внимания больше одного два человека.
2)Соавторство с другим математиком, сомнительный путь требующий 500+ харизмы.
3)Написать псевдо-научно-популярные бредни в стиле ruwiki://Великая_тайна_воды ну а дальше пипл-схавает.
>> No.74994 Reply
>>74993
Как-то недостаточно конкретики. Я ещё раз отмечу6 прошу не отвечать тех, у кого нет своих публикаций.
>> No.74996 Reply
>>74994
Начни читать книжки которые рекомендовали в этом треде, в процессе чтения разберешься какая часть математики тебе интересна, дальше реквестируй новые книжки в этом треде, на каком-то этапе лучше завести знакомого математика который мог бы тебя консультировать. Достаточно углубившись в своей теме ты сможешь что-нибудь доказать/описать, из не описанного ранее, вот и выйдет твоя статья, дальше дело за малым отправить ее в какой-нибудь журнал.
Учитывай что это займет не один и не два года, вероятность успешного исхода для тебя < 0.01.
И вообще ты спрашиваешь что-то в духе "как стать ученым" ответ - кучу времени, желательно со старшей школы изучать выбранный тобой предмет.
>> No.75012 Reply
>>74994
Начинай читать ВАКовские журналы, поймёшь, о чём там пишут. Потом накатаешь свою работу и отправишь в редакцию какого-нибудь из них.
http://vak.ed.gov.ru/ru/help_desk/list/
>> No.75013 Reply
>>74757
>>74774
Большое спасибо!
>> No.75020 Reply
>>75013
Всегда пожалуйста. :3
>> No.75025 Reply
>>74980
Поступай в государственный ВУЗ твоего города на мехмат (матмех, матфак), специализируйся в топологии, оканчивай бак, оканчивай магу. На выходе из маги у тебя будет статья и не одна, инфа 100%, при этом ты будешь полноправным белым человеком, занимающимся наукой.
Итого: тебе для этого нужно 6 лет, учитывая, что поступление в этом году ты проебал, то уже 7. Оно тебе надо? Другие пути мне неизвестны. Учитывай также, что никакая редакция более менее котируемого нашего или забугорного журнала не примет твою статью без рецензии какого-нибудь хуя.
>> No.75067 Reply
File: big_1339270585teaser.jpg
Jpg, 40.36 KB, 644×390 - Click the image to expand
edit Find source with google Find source with iqdb
big_1339270585teaser.jpg
Посоны, дано несколько точек на плоскости, требуется соединить их минимальным путем.
>> No.75070 Reply
>>75067
Соединяй любым путем. Это будет путь, минимальный по умственным усилиям.
>> No.75072 Reply
>>75067
Волновой алгоритм. Это не к нам.
>> No.75074 Reply
>>75070
Ок, путем минимальной длины.
>>75072
Не оно. Там ребра единичной длины.
>> No.75075 Reply
>>75074
Минимальное расстояние между двумя точками - отрезок, лежащий на прямой. Длина отрезка между точками с координатами A(a,b) и B(c,d) рассчитывается как AB = sqrt((c-a)^2 + (d-b)^2), где sqrt - квадратный корень, ^2 - возведение в квадрат.
>> No.75077 Reply
>>75075
А если точек больше двух?
>> No.75078 Reply
>>75067
Когда решишь, расскажешь мне в личку.
мимо-коммивояжер
>> No.75080 Reply
>>75078
проиграл
>>75067
погугли задачу коммивояжера
>> No.75081 Reply
>>75080
Я знаю о задаче коммивояжера. Это разные задачи.
>> No.75082 Reply
>>75081
Это частный ее случай.
>> No.75083 Reply
File: Квадрат.png
Png, 9.21 KB, 320×320 - Click the image to expand
edit Find source with google Find source with iqdb
Квадрат.png
>>75082
Кажется мне, что ты не понял мою задачу. Ну-ка реши для вершин квадрата.
>> No.75112 Reply
>>74787
К сожалению, сюда. Здесь много реалистов и почти нет математиков.
>> No.75132 Reply
>>75067
Ну алгоритмически она решается элементарно, считаешь квадраты расстояний и перебираешь каждый с каждым. Математически будет сложнее, по любому придётся составлять какую-то матрицу расстояний, минимизировать функцию ошибки. Как вариант, погугли методы кластеризации, может из них что-то cкомбинируешь.
>> No.75139 Reply
>>75132
> Ну алгоритмически она решается элементарно, считаешь квадраты расстояний и перебираешь каждый с каждым.
Во-первых, не решается она так, ибо не факт, что между данными точками соединения будут прямыми.
Во-вторых, твой алгоритм для n точек будет перебирать (n-1)! путей, что для больших n будет решаться годами в лучшем случае.
>> No.75161 Reply
Глупый вопрос, но я не могу иначе:
Аноны, вы когда читаете математику, какие-нибудь сопутствующие задания прорешиваете? Или просто запоминаете материал?
И ещё: чтобы понять самый «крутой» материал — надо быть гением или чем-то вроде него?
>> No.75170 Reply
>>75161
> чтобы понять самый «крутой» материал — надо быть гением или чем-то вроде него?
Нет, просто знать используемые определения и теоремы.
>> No.75241 Reply
>>74216
Начал читать Ван дер Вардена. Это божественно.
>> No.75248 Reply
>>75161
Смотря что понимать под читаете. Если я читаю научную статью, то я стараюсь понять доказательства, которые в ней есть, попутно почитывая сопутствующий материал, чтобы быть больше в теме.

Если же я читаю книгу, то я стараюсь уловить суть каждого раздела/параграфа, в доказательствах разбираюсь редко, ибо неохота, да и топология такая наука, что там можно весь день разбираться в доказательстве на страницу, на задачки/упражнения поглядываю, если есть интересные на мой взгляд, то сажусь и загружаюсь (такие обычно помечаются звёздочкой).
> И ещё: чтобы понять самый «крутой» материал — надо быть гением или чем-то вроде него?
Усердным гением.
>> No.75256 Reply
>>75161
> задания прорешиваете
Нет.
> просто запоминаете материал
Тоже нет.
> чтобы понять самый «крутой» материал — надо быть гением или чем-то вроде него?
Достаточно просто мыслить как математик.
>> No.75261 Reply
>>75161
> сопутствующие задания прорешиваете?
Без этого чтение учебника (речь ведь об учебниках, как я понял) не имеет смысла: появится иллюзия понимания, которая не имеет ничего общего с самим пониманием. Математика заключается в деталях и тонкостях, которые проявляются в задачах.
>> No.75270 Reply
>>75170
>>75256
>>75261
Спасибо.

>>75256
> Достаточно просто мыслить как математик.
Если долго изучать математику, пытаться найти нетривиальные решения и прочее: тогда и математическое мышление придёт?
>> No.75280 Reply
ftp://ftp.mccme.ru/users/prasolov/geometry/book.pdf
Как анон относится к этой книжке?
Я начал читать, и пока что фанатею,
будь такая книга в моём детстве,
я б выучил геометрию годам к семи.
>> No.75281 Reply
>>75280
Анон её одобряет люто-бешено, как и все книжки Независимого Московского Университета.
http://ium.mccme.ru/idx.html
http://lurkmore.to/НМУ
>> No.75282 Reply
>>75075
Меня твоя евклидова метрика на R^2 доебала уже просто.
Координаты какие-то, хуйня, малафья.
>> No.75283 Reply
File: x_db902220.jpg
Jpg, 146.43 KB, 600×486 - Click the image to expand
edit Find source with google Find source with iqdb
x_db902220.jpg
>> No.75284 Reply
>>75281
Я знаю про НМУ, конечно, я ж туда собираюсь с сентября пойти.
Кстати, есть ИТТ аноны из НМУ?
>> No.75286 Reply
>>74787
Представь, например, четырёхмерный шар как эволюцию его сечения трехмерной гиперплоскостью.
То есть, представь, что через трехмерную гиперплоскость, где ты живёшь, продавливают 4-мерный шар, а ты видишь его сечения.
Точка - увеличивающийся шар - уменьшающийся шар - точка.
Потом подумай, как строятся многогранники, на примере куба того же. Квадрат (параллелограмм) (2-мерный) на евклидовой плоскости ограничивается двумя парами параллельных прямых (одномерных). Куб (параллелепипед) (3-мерный) ограничивается тремя парами параллельных плоскосей (2-мерных). Подумай, чем ограничивается 4-куб, какие у него 3-мерные грани, что есть пересечение этих граней, представь, как выглядит трехмерная развёртка 4-куба (нарисуй).
Подумай, можно ли разомкнуть целые, замкнутые в R^3 кольца, в R^4.
Смоги это, а представление само разовьётся постепенно.
>> No.75287 Reply
>>75012
Нахуя читать, чукча не читатель же.
>>74994
Вот:
ruwiki://SCIgen
и не слушай всяких учёных-кипячёных, Корчеватель юбер аллес.
Привет от Рамзана.
>> No.75303 Reply
>>75284
ходил полгода на первый курс, когда сам уже был на четвертом физтеха. Спрашивай вопросы, если етсь.
>> No.75306 Reply
>>74787
Начни с пятимерного. Нечетномерные представить проще.
>> No.75313 Reply
> Я бы поискал обзорную статью, где сильнейший из ныне живущих аналитиков подробно объясняет, что есть математический анализ
Perspectives on geometric analysis? Или что-то другое?
>> No.75317 Reply
>>75313
оно самое
>> No.75318 Reply
>>75284
> Кстати, есть ИТТ аноны из НМУ?
А то!
Спрашивай советов мудрых, не стесняйся.
> Я знаю про НМУ, конечно, я ж туда собираюсь с сентября пойти.
Видел уже расписание? В среду первое занятие -- геометрия. Тебе очень повезло с преподами по этому предмету, кстати.

Я просто приведу для тебя одну цитату.
Молодых юношей, желавших вступить в орден, держали перед закрытыми воротами от нескольких суток до нескольких недель.
На первое занятие придёт 200-300 человек и ты скорее всего не влезешь в аудиторию, лол
Только самых настойчивых приглашали во внутренний двор. Там их заставляли несколько дней впроголодь у тебя не всегда будет хватать времени на сон и еду сидеть на холодном каменном полу, довольствуясь скудными остатками пищи и ждать, порой под ледяным проливным дождем или снегопадом, когда их пригласят войти внутрь дома.
Время от времени на внутреннем дворе перед домом Ибн Саббаха появлялись его адепты из числа прошедших первую степень посвящения. Они всячески оскорбляли, даже избивали молодых людей, самодовольные взгляды матшкольников желая проверить, насколько сильно и непоколебимо их желание вступить в ряды хашшашинов. В любой момент молодому человеку позволялось подняться и уйти восвояси В НМУ свободный вход, но самое главное -- из НМУ свободный выход, лол.
Лишь прошедшие первый круг испытаний допускались в дом Великого Владыки. Их кормили, отмывали, переодевали в добротную, теплую одежду… Для них начинали приоткрывать «врата иной жизни».

Вообще, если ты раньше никогда не сдавал листочки незнакомым людям, не ходил в кружки и не участвовал в матшкольных тусовках, то первые лекция и семинар в НМУ это очень специфический опыт.
Желаю тебе слиться не раньше Ноября :3
>> No.75319 Reply
Раз уж пошла такая хуйня, насоветуйте годных математических журналов, что ли. Лучше, конечно, на английском.
>> No.75321 Reply
Почитал тред.
> Математики, которая не может быть к чему-нибудь приложена, вообще быть не может.
> Арнольд то, Арноль сё
> математическая физика
> Тривиум
Ок, ок, всё здорово. Только какого хуя тред называется "Кафедра математики"? Тут есть вообще хоть кто-то, кто имеет к математике хоть какое-то отношение, или только фанаты Арнольда?
>> No.75329 Reply
File: gendo4.jpg
Jpg, 9.93 KB, 300×207 - Click the image to expand
edit Find source with google Find source with iqdb
gendo4.jpg
>>75321
Что есть математика? Кто есть математики?
>> No.75333 Reply
>>75319
http://www.springerlink.com/content/0037-4466/
Там и мои даже есть, хе-хе :3
>> No.75338 Reply
File: Vladimir_Voevodsky.jpg
Jpg, 11.83 KB, 220×186 - Click the image to expand
edit Find source with google Find source with iqdb
Vladimir_Voevodsky.jpg
Привет аноны, добавлю своих рекомендаций. Я undergrad в западном вузе. (inb4 Мне 20 лет. Я работаю в компании-лидере в своей отрасли...) Первая и самая важная рекомендация: учиться и писать статьи нужно на английском, так как в рососсии все очень хуево. Можно смотреть лекции с винрарного сайта ocw.mit.edu и спрашивать если что непонятно на math.stackexchange.com, ходить на лекции НМУ если в дефолт сити или пинать хуй если не. про этот доброчан лучше забыть, тут в основном хуи какие-то для флейма и троллинга собрались. Рекомендованные книги почти все на английском. Возможно, кому-то из вас мой взгляд покажется несколько однобоким, ну так сосите хуй. Остальные, спрашивайте свои ответы. Знаком (!) отмечены книги которые читать можно практически с нуля. Ну поехали блядь:

Математический анализ - (!) Rudin: Principles Of Mathematical Analysis, первая и единственная
Алгебра - (!) Aluffi: Algebra: Chapter 0, это просто божественное откровение а не книга, все объясняется современно и на языке категорий, который вводится по ходу дела. А, кстати, тут много мудаков рекомендующих Ван дер Вердена, вам в музей вместо с этим старьем. После этой книги надо смотреть по интересам, например можно изучить коммутативную алгебру по Atiyah, Macdonald: Introduction To Commutative Algebra, книга охуенна и является пререквизитом к алгебраической геометрии в толковании Хартсхорна. А вообще очень хорошо бы изучить гомологическую алгебру, после Алуффи рекомендую почитать публикацию Гротендика в Тохоку, если че то я ее сюда выложу, но вроде в интернетах есть. Также хвалят текст Peter Hilton; Stammbach, U. A course in homological algebra, но я сам его не читал.
Аналитическая геометрия - это еще блядь что такое
Линейная алгебра и геометрия - (!) Axler: Linear algebra done right, вряд ли нужно знать больше того, о чем рассказоно в этом тексте.
Дискретная математика - Учебников хороших не знаю, но вот например лекции http://www.math.harvard.edu/~lurie/155.html и задачки там же.
Математическая логика - (!) Azriel Levy: Basic Set Theory, потом The Handbook Of Set Theory
Дифференциальная геометрия - (!) Manfredo do carmo: Differential Geometry
Топология - многие советуют Munkres и сразу идут на хуй. Для начала стоит прочитать (!) Klaus Janich: Topology, после нее можно углубиться в другие разделы топологии: для алгебраической лучше всего подходит Allen Hatcher: Algebraic Topology. По К-теории стандартная рекомендация это M. Atiyah: K-theory, ну еще можно почитать A Hatcher. Vector Bundles and K–Theory, но она вроде все еще не закончена. Если еще что по топологии интересно то пешите посоны, есть дохуя годных книг.
Теория функций комплексного переменного - Cartan: Elementary Theory of Analytic Functions of One or Several Complex
Теория вероятностей - (!) Grimmet, Welsh: Probability, an introduction
Категории - MacLane: Categories for working mathematician
Алгебраическая геометрия: эта дисциплина требует кучу пререквизитов, кто-то выложил Майлза Рида, но у него стиль специфический и непонятно ничего. Самый легкий в плане пререквизитов это Hartshorne: Algebraic Geometry, нужно знать алгебру и топологию в пределах первых глав книжек отмеченных (!) выше, еще коммутативную алгебру по Атье, желательно пройти всю или придется много брать на веру. Есть еще Griffiths, Harris: Principles of Algebraic Geometry, к которой дохуя пререквизитов но она того стоит.

Приведенные книги дадут начальный уровень понимания, на полное изучение всего понадобится года три-четыре, в зависимости от нагрузки, такие дела.

Удачи всем.
>> No.75360 Reply
>>75318
Я смотрел программу и листочки прошлых лет, и мне кажется, что первый семестр очень простой, могу хоть сейчас экзамены сдавать.
> cдавал листочки незнакомым людям
Чем оно принципиально отличается от сдачи экзамена/зачёта незнакомому преподавателю?
Про геометрию. У Сосинского какая-то странная программа для первого семестра, напичканная всякими красивыми и интересными штуками, но неупорядоченная, по-моему. Хотя я был уже на одной лекции той осенью (про coxeter geometries, мимикрокодил), было интересно и вполне понятно. Каково ваше отношение к программе Сосинского?
А ещё вопрос про пятую пару.
Ничего страшного, если по одному из предметов приходить только на семинары?
На лекцию можно успеть, дай бг, с четвёртой пары (из МГУ).
А пятая пара у меня в один из {Пн, Ср, Пт} будет обязательно.

Кстати, почему-то в этом году нет оргсобрания в начале сентября.
Что там обычно происходит, ничего особенного ведь, я так понимаю.
И что происходит на семинарах? Просто сдаются задачи из листков (устно), или что-то ещё?
>> No.75362 Reply
>>75319
Удваиваю вопрос. Из того, что я знаю: Bulletin of AMS, Notices of AMS.
http://www.mathnet.ru/php/archive.phtml?jrnid=intf&wshow=contents
http://www.mathnet.ru/php/journal.phtml?jrnid=rm&option_lang=rus - эти два на русском, но хорошие.
>> No.75365 Reply
>>75360
> Чем оно принципиально отличается от сдачи экзамена/зачёта незнакомому преподавателю?
Ничем, конечно. Я просто первый раз боялся, что мне зададут вопрос, на который я не смогу ответить, или от волнения скажу какую-нибудь глупость. Для таких случаев есть простая фраза "давайте следующую", лол.
> Про геометрию.
Я прохладно отношусь ко всякой бижутерии, которой много в программе Сосинского. Он утончённый человек и ему хотелось побольше всяких красивостей, но время можно распределить более практично, как мне кажется. Вообще, содержание предмета "геометрия" первого семестра всегда и у всех разное.
> Ничего страшного, если по одному из предметов приходить только на семинары?
???
Так это вообще никого не волнует. Тебе нужно сдать определённое количество задач и написать экзамен, а как ты будешь распределять время -- исключительно твой персональный вопрос.
Поделюсь собственным опытом: "задачи > чтение книжек". Когда я слишком подробно разбирал теорию, а задачи оставлял на потом, всегда приходилось туго.
(На самом деле это проявление лени: пассивно читать книжки гораздо проще, чем решать задачи)
> почему-то в этом году нет оргсобрания
> Что там обычно происходит
Представляют преподавателей 1го курса, объясняют принципы обучения, которые итак подробно описаны на сайте, отвечают на вопросы, ответы на которые можно найти на сайте и т.д.
Думаю, даже меньшая часть желающих просто не влезла бы в конференц-зал, поэтому нет смысла проводить.
> И что происходит на семинарах?
Устная сдача задач (хотя, сдаётся мне, на геометрии вам разрешат сдавать письменно, но устно проще же намного).
Шум, обсуждения, кипиш, возня, очередь к преподам.
>> No.75366 Reply
>>75318
Доброанон, а есть ли смысл идти чисто на первый курс, чтобы подтянуть матан (ну и алгебру тоже бы не мешало)?
Учусь на прикладного в магистратуре. Плохо понимаю оптимизацию, а работать с зазубренным инструкциями вида "делай так - получишь максимум" - нахуй так жить. Да и вообще, такая деятельность, как софтваре инжиниринг вызывает у меня некоторые отторжение. Мне бы хотелось, чтобы когда-нибудь в моей работе было чуть больше математики и чуть меньше программирования.
Я думаю, что НМУ он все-таки немного для других целей и уж точно не для прикладников, но что делать решительно не знаю. Больше не вижу мест, где можно найти помощь в освоении, никто либо сам не знает, либо время на меня тратить не будет.
>> No.75370 Reply
>>75365
> ко всякой бижутерии, которой много в программе Сосинского
Я вот так примерно и понял, да.
> Я просто первый раз боялся, что мне зададут вопрос, на который я не смогу ответить, или от волнения скажу какую-нибудь глупость.
Ты, небось, сразу пошле школы пошёл.
Меня как раз гнетёт то, что я не могу ни с кем обсудить то, что я выучил/освоил/решил, потому что есть страх, что я что-то делаю неправильно или нерационально, а сам этого никогда не увижу.
А быстро ли сливаются все эти 200-300 человек, которые приходят на первые занятия? Оче неохота стоять в колоссальных очередях, чтобы сдать задачи.
Да, и записываться и заполнять анкету мне необязательно сейчас, я так понимаю.
Оно ведь имеет смысл только когда сдашь три экзамена и станешь полноправным слушателем, или нет?
>> No.75372 Reply
>>75366
Смысл есть, но сделать это трудно, потому что будет оче много людей.
> Больше не вижу мест
ШАД? Но там, конечно, экзамены вступительные.

>>75370
> есть страх, что я что-то делаю неправильно или нерационально, а сам этого никогда не увижу
именно для этого и существует сдача задач. Цель не просто в том, чтобы решить, а чтобы убедиться, что решил правильно.
> А быстро ли сливаются все эти 200-300 человек
Думаю, где-то в Ноябре появятся свободные места в конференц-зале.
К концу 1го курса остаётся человек 20 посещающих (хотя как-то поздней весной я видел лекцию 1го курса, на которой присутствовало 3 человека, лол).
> записываться и заполнять анкету мне необязательно сейчас
Это нужно, думаю, организаторам, чтобы знать, сколько придёт народу. В прошлом учебном году размещали остатки в столовой, боюсь, в этом году могут и не уместиться. Мне даже интересно.
> станешь полноправным слушателем
Студент от вольнослушателя отличается только микростипендией и какими-то дополнительными правами в библиотеке.
>> No.75374 Reply
>>75370
Сливаются относительно быстро. Через одну-две недели все помещаются в двух аудиториях. Месяца через два - в одной.
А лекции записываются на видео и выкладываются на сайте.

Алсо, доброанон-геометр, ты случайно не относишься ли негативно к комитету премии Абеля? :3
>> No.75375 Reply
Анон, не могу выполнить задание, вернее не понимаю как:
Является ли функция дифференцируемой в точке (0,0)
Там какие должны быть примеры? Если у меня функция (8x^3 -y^3)^1/3 (корень короче) Там через пределы или как.

Алсо реквестирую книгу по комбинаторике.
>> No.75376 Reply
>>75372
> Студент от вольнослушателя отличается только микростипендией и какими-то дополнительными правами в библиотеке.
По-моему, ещё правом посещать занятия по ин. языкам.
А сколько стипендия? (просто интересно)
>> No.75378 Reply
>>75375
Предел справа должен быть конечен и равен пределу слева, этого же достаточно, вроде.
>> No.75379 Reply
>>75374
> не относишься ли негативно к комитету премии Абеля?
Атья, Громов, Милнор и тут, блядь, Семереди? :3
>> No.75380 Reply
>>75375
Не, я тупой, здесь, скорее всего, нужно показать существование (необходимо) и непрерывность (достаточно) частных производных в (0;0).
>> No.75387 Reply
>>75378
Это не то, но спасибо за ответ. Уже решил, там через ро и приращения. И то что в конце там предел существует.
>> No.75390 Reply
>>75318
Первое занятие на первом курсе - анализ, в пятницу 7 сентября.
>> No.75405 Reply
>>74216
Аноны, накидайте, пожалуйста, годных книг по мат. логике. Только нужно с самых-самых азов что бы всё.
>> No.75419 Reply
Анон, а за сколько примерно нужно приходить в НМУ на первое занятие, чтобы в аудиторию попасть?
>> No.75432 Reply
File: DSCN0577.JPG
Jpg, 619.32 KB, 1789×1600 - Click the image to expand
edit Find source with google Find source with iqdb
DSCN0577.JPG
Анон, я уже реально потерял надежду, никак не пойму пикрелейтед, какую площадь мне надо построить, какая эта область? Преподователь в начале июля дал имена авторов где можно посмотреть по площадям, но я криво записал и в каракулях не мог найти учебник, а во всяких зоричах с ильинами я не пойму. Вот, уравнение обведено же.
>> No.75434 Reply
>>75432
> площадь мне надо построить
А можно понятней? Что такое "построить площадь"? В чём суть задачи?
Не понял, пределы интегрирования от двух до чего? Там какая-то загогулина, из контекста на пикрелейтед непонятно.
>> No.75442 Reply
>>75434
От альфы до беты. Нарисовать график к этой функции, я уже сам не понимаю чего от меня хотят, в типовом расчете про такие вещи ни слова. Это дополнительное задание.
>> No.75444 Reply
>>75442
Извини, но я вот тоже нихуя не понял.
Видимо, спрашивается про пределы интегрирования.
Хорошо бы полное условие задачи.
>> No.75446 Reply
>> No.75473 Reply
>>75446
Всё понял.
Поменяй в формуле местами параметрические функции, там будет производная от 32 косинусов в кубе. (даже пруф нашёл тебе: http://www.cleverstudents.ru/area_of_parametrically_defined_figures.html)
Пределы интегрирования возьми от 0 до pi/3, а результат потом удвой лучше.
>> No.75474 Reply
>>75446
Точнее, от pi/3 до нуля, по-моему, там как по знаку получится.
>> No.75510 Reply
>>74216
Аноны, увидел фразу "тензор - это сечение расслоения". Скажите пожалуйста, что такое расслоение?
>> No.75511 Reply
File: FiberBundle-01.png
Png, 1.89 KB, 145×121
edit Find source with google Find source with iqdb
FiberBundle-01.png
File: marijuana_and_mat...
Jpg, 173.34 KB, 736×536
edit Find source with google Find source with iqdb
marijuana_and_math.jpg

>>75510
1. Локально-тривиальное расслоение топологического пространства E над пространством B со слоем F -- это такое сюръективное отображение E на В, что у каждой точки из В есть такая окрестность U, что прообраз этой окрестности гомеоморфен прямому произведению UxF, причём этот гомеоморфизм, который называется тривиализацией, в композиции с проекцией на первый сомножитель совпадает с исходным отображением (см. картинку №1)
2. Элементарный пример: проекция квадрата на одну из сторон -- расслоение над отрезком со слоем отрезом.
3. В каждой точке гладкого многообразия определено касательное пространство. Совокупность касательных пространств во всех точках снова является гладким многообразием вдвое большей размерности. 0тображение, которое каждому касательному пространству сопоставляет точку, в которой оно приложено, является расслоением, слой которого -- векторное пространство. Это отображение называется касательным расслоением.
4. Сечением расслоения называется отображение (непрерывное, конечно, всё непрерывное и нужное кол-во раз гладкое) из B обратно наверх в E такое, что его композиция с отображением расслоения является тождественным.
Грубо говоря, сечение каждой точке из B даёт один элемент из слоя F.
5. Пример: любое векторное поле на многообразии это сечение касательного расслоения.
6. Векторное расслоение -- расслоение, слой которого -- векторное пространство (а переходы между тривиализациями -- линейные изоморфизмы). Пример векторного расслоения -- указанное выше касательное расслоение.
С векторными расслоениями можно делать всё то же самое, что и с векторными пространствами, брать тензорные/симметрические/внешние степени, переходить к двойственному пространству и т.д.
7. Примеры: дифференциальные 1-формы -- сечения расслоения, двойственного к касательному. Дифференциальные n-формы -- сечения n-й внешней степени этого (оно называется кокасательное) расслоения.
8. Такие образом любое тензорное поле на многообразии является сечением какой-то тензорной/внешней/симметрической степени (ко)касательного расслоения.
9. ???
10. См. картинку №2
>> No.75513 Reply
>>75375
Композиция непрерывных функций непрерывна.
>> No.75518 Reply
>>75513
Пруф?
>> No.75520 Reply
>> No.75521 Reply
>>75518

Вообще, мне кажется, что это очевидно.
Доказательство: просто 2 раза применяем определение непрерывной функции.
Сначала в точке f(x) для g(), потом в точке x для f => g(f()) также непрерывна в x.
>> No.75525 Reply
>>74482
Андрюша, говнюк, ты ли это?
>> No.75551 Reply
А кто-нибудь может привести пример Евклидового кольца и пары элементов в нём, чтобы деление можно было провести разными способами?
(однозначность деления не входит в определение, а Винберг, например, это даже подчёркивает)
>> No.75556 Reply
>>75551
Целые подходят:
f(x) = |x|
3 = 2*1 + 1 = 2*2 - 1
Алсо, разложение на неприводимые множители в евклидовых однозначно.
>> No.75557 Reply
>>75556
самые простые примеры от меня ускользают :3
Я уже нашёл такую пару в числах Эйзенштейна.
>> No.75605 Reply
передумал я в ваш нму идти, это не торт приезжать домой в 11+
>> No.75608 Reply
>>75605
Это ты передумал, а я не передумал.
Тем лучше, меньше народа - больше кислорода.
Я люблю трэш-угар и поздние электрички, а ты не тру.
Капча: прозреваю гомофобию выплевывая богатого.
>> No.75620 Reply
>>75608
Ты сам забьешь, ибо ты не знаешь какие нагрузки, если работаешь, учишться в инсте, да еще нму.
Вот что ты днем делаешь?
>> No.75643 Reply
>>75620
> если работаешь
Работа не нужна, очевидно же. Только чистые знания, только жить на ренту.
>> No.75658 Reply
>>75643
> Только чистые знания, только жить на ренту.
Не знаю, ирония это или нет, но многие действительно так думают.
И я в том числе
>> No.75704 Reply
>>75620
Нагрузки - это одно.
А НМУ - это не нагрузки, а отдых.
Я один хуй раньше 22:30 домой редко приезжаю.
>> No.75715 Reply
>>75704
> А НМУ - это не нагрузки, а отдых.
Ты ебнутый.
МимиНМУшник
>> No.75719 Reply
НМУ не нужен.
>> No.75721 Reply
>>75715
> Ты ебнутый.
Будто что-то плохое.
>> No.75724 Reply
Перепись wannabe-нмушников стартует прямо сейчас:
пишите дневное место учёбы, на какие курсы собираетесь ходить, сколько времени планируете продержаться и т.д.
>> No.75725 Reply
>>75724
Ага, и давайте еще уютный чатик сделаем, а на первое занятие белые браслеты наденем.
>> No.75734 Reply
>>75719
Нет, ты.
>> No.75736 Reply
По поводу алгебры. Прочитал Винберга. Ван дер Варден основательнее и, имхо, аккуратнее.
> Aluffi: Algebra: Chapter 0
http://www.mimuw.edu.pl/~jarekw/pdf/Algebra0TextboookAluffi.pdf - оно? Бегло просмотрел. Тот же Варден, только в профиль.

Пошёл читать "Теорию операторов" Садовничего, "Теорию матриц" Гантмахера и "Дифференциальную топологию" Милнора-Уоллеса.
составитель_программы-кун
>> No.75737 Reply
>>75736
Как ты собираешься это все использовать?
>> No.75744 Reply
>>75338
> Дифференциальная геометрия - (!) Manfredo do carmo: Differential Geometry
Стоящая книжка?
Во всех списках обычно фигурирует что-нибудь из J.M.Lee в компании с Helgason: Differential geometry, Lie groups, and symmetric spaces
или что-нибудь доисторическое типа Kobayashi and Nomizu
>> No.75746 Reply
>>75744
Старое - не значит плохое.
>> No.75747 Reply
>>75746
Сам я в Кобаяси-Номидзу особо не вникал, но кто пробовал говорили, что как-то перегружено, и есть более удачные изложения.
>> No.76051 Reply
Привет, доброаноны.
Запрашиваю чтив по выпуклому анализу, оче нужно.
>> No.76068 Reply
>>75280
Просто по российской фамилии могу априорно говорить, что это не математика.
>>75338
Про Варден Ван Дера согласен, это для школьника или технаря. Леви у тебя видел, наверное, неплохо.
Ты говоришь о математической логике как о математике, а не как об области философии, поэтому твои рекомендации по этому предмету я критикую отрицательно.
>> No.76070 Reply
>> No.76078 Reply
>>75280
Не имеет эта книга отношения к математике.
>>76068-кун
>> No.76081 Reply
>>76078
А ты имеешь какое-нибудь отношение к математике?
>> No.76082 Reply
>>76081
Я учусь на мехмате, вот мой блог:
http://justmylegend.blog.ru/
А в вандервардене даже про интегрирование по частям нет, я содержание посмотрел.
>> No.76083 Reply
>>76082
Ай, блять, жалко он всю вкусняшку из блога потёр.
>> No.76084 Reply
>>76082
> А в вандервардене даже про интегрирование по частям нет, я содержание посмотрел.
Но это же не учебник по матану. Или я чего-то не понимаю в алгебре
>> No.76085 Reply
Порекомендуй же что-нибудь по мат. логике.
>> No.76086 Reply
>>76084
Это петросян приходил.
>> No.76090 Reply
File: kabala1.gif
Gif, 29.56 KB, 480×703 - Click the image to expand
edit Find source with google Find source with iqdb
kabala1.gif
>>75338
> на полное изучение всего
> понадобится года три-четыре
> на изучение всего
> полное
>> No.76112 Reply
>>76082
> Я учусь на мехмате
Это не на том ли, где "тензор - это набор чисел, который при замене координат преобразуется по правилу..."?
> Просто по российской фамилии могу априорно говорить, что это не математика.
Читал ли ты книгу дальше фамилии автора, мой юный друг?
>> No.76131 Reply
>>76082
Лол, это же тот хуй, которого Вербицкий обсирал.
Это действительно ты, или шутник дохуя.
>> No.76132 Reply
Как мне противно стало от вашего треда, сразу повеяло запахом Арнольда, школьников, тусклых фонарей по дороге с вечернего маткружка, студентов-ассистентов, вызывающе некрасивых девушек, хихикания конченных задротов, обсуждения недавнего похода в Хибины, летней матшколы, листочков Константинова, бородатых фриков из МЦНМО, споров на тему 2>57, случайно прибившихся нормальных парней из "обычных" щкол, тихоней в свитерах, вращающихся дверей ГЗ, бесцельных обсуждений какой-то задачи, чая с мерзким печеньем, тонких книжек в качестве приза на олимпиаде, пинпонга, заверений что деньги по жизни не нужны и они есть только у быдла.
Мимоолдфаг
>> No.76133 Reply
>>76132
Последнее предложение недописал, а зря.
>> No.76138 Reply
>>76112
Сударь, это мимотролль с тифаретника. Можете не обращать на него внимания.
>> No.76145 Reply
Посоветуйте книжку по дифурам, чтобы даже дурак мог разобраться, с примерами и понятно.
>> No.76153 Reply
>>76145
Обыкновенные дифференциальные уравнения, Арнольд.
http://rghost.ru/40176102
>> No.76162 Reply
>>76081
У меня есть уверенная позиция и подход.
Алсо, >>76082 - не я, и на вопросы, заданные ему, я отвечать не должен, также не должен обращать внимания на выпады в его сторону.
>> No.76188 Reply
Cписок достойных русскоязычных книг по алгебре:
Ван дер Варден, "Алгебра"
Винберг Э.Б., "Курс алгебры"
Ленг, "Алгебра"
Гельфанд И.М., "Лекции по линейной алгебре"
Кострикин, "Введение в алгебру"
Кострикин-Манин, "Линейная алгебра и геометрия"
Курош, "Курс высшей алгебры"
Курош, "Общая алгебра"
Ильин-Позняк, "Линейная алгебра"
Беклемишев, "Курс аналитической геометрии и линейной алгебры"
Ефимов, Розендорн, "Линейная алгебра и многомерная геометрия"
Воеводин, "Линейная алгебра"
Головина, "Линейная алгебра и некоторые ее приложения"

Я кого-нибудь забыл?
>> No.76197 Reply
>>76188
Ну почему Вандерварден-то?
Не, ну он, может, и хороший, но мезозоем попахивает же.
>> No.76199 Reply
>>76197
Он фундаментальный. Все остальные авторы копипастят его с небольшими дополнениями.
>> No.76201 Reply
Первокурсник врывается в тред. Вы не поверите, что нам задали по матану после первой лекции. Доказать, что декартово произведение двух окружностей - это тор. Блджад, я представляю себе этот бублик, но я даже представить себе не могу, каким образом он может получиться из произведения двух окружностей. По моим представлением для этого нам понадобится четвертое измерение или два литра водки
>> No.76206 Reply
>>76201
> Доказать, что декартово произведение двух окружностей - это тор
Так это определение тора.
А вообще, лучше правда пей водку, до сессии ещё далеко, успеешь всё заботать :3
>> No.76208 Reply
>>76201
Таки там действительно 4е измерение нужно, и это действительно определение тора, или у вас оно другое?
>> No.76211 Reply
>>76206
>>76208
Нам просто объяснили, что такое декартово произведение (в двух словах), а потом сразу же дали это задание на дом (ну и перед этим еще декарт. произведение двух отрезков и окружности с отрезком, но с этим я разобрался) с указанием, мол, если что, это фигура называется "тор". Всё.
>> No.76213 Reply
>>76211
Непонял.
> Доказать, что декартово произведение двух окружностей - это тор.
> с указанием, мол, если что, это фигура называется "тор".
Так доказать или представить как выглядит?
>> No.76214 Reply
>>76213
Задание - доказать. Но мне бы для начала это хотя бы представить.
>> No.76222 Reply
File: Тор2.jpg
Jpg, 9.99 KB, 225×225
edit Find source with google Find source with iqdb
Тор2.jpg
File: Тор.jpg
Jpg, 7.59 KB, 300×200
edit Find source with google Find source with iqdb
Тор.jpg

>>76201
Круг - это множество точек, расстояние которых от центра круга не превышает радиуса. Круг можно себе представить как диск.
Окружность - это граница круга. Можно представить как обруч.

Возьмём обруч и каждой его точке сопоставим некоторый круг (наденем на обруч диск и провернём). Получится множество дисков, надетых на обруч. Это - тор. Иными словами, тор - это декартово произведение окружности на круг.

Если надеть на обруч обручи, то получим торическую поверхность. Тот же бублик, только пустой внутри. Иными словами, торическая поверхность - произведение окружности на окружность.

Задача. Подумай, как ввести координаты на окружности (подсказка: числовая окружность), на круге, на поверхности цилиндра, на цилиндре, на поверхности тора, на торе.
>> No.76223 Reply
>>76201
Множество - это совокупность элементов контейнер, в котором что-то лежит. Множество - это фундаментальное понятие, не определяемое через другие.
Пустое множество - множество, не содержащее элементов.
Элементами множества могут быть другие множества и вообще всё, что угодно.

Декартовым произведением двух множеств A и B называется множество AxB, составленное из всевозможных упорядоченных пар <a,b>, где a принадлежит A, b принадлежит B.
Пример 1: A = {1,2,3}, B = {Кира, L},
AxB = { {1, Кира}, {1, L}, {2, Кира}, {2, L}, {3, Кира}, {3, L} }
Замечание. <3, Кира> и <Кира, 3> - это разные кортежи. Порядок важен.

Понятие декартова произведения естественным образом обобщается на n множеств.
Пример 2: A = {a,b}, B = {яблоко, груша}, C = {Хоро}
AxBxC = {{a, яблоко, Хоро}, {a, груша, Хоро}, {b, яблоко, Хоро}, {b, груша, Хоро} }

Элементы декартова произведения n множеств называются n-местными кортежами (кортежами длины n) или упорядоченными энками (двойками, тройками, четвёрками и так далее)

n-местным отношением на множествах A1, A2, ... , An называется некоторое подмножество декартова произведения A1 x A2 x ... x An. Если n=1, то отношение называется унарным, если n=2, то отношение называется бинарным, если n = 3, то отношение называется тернарным. Отношения обычно обозначаются {ф: A x B} или AфB.

Пусть имеем множество A и множество B. Два элемента a из A и b из B называются находящимся в отношении AфB, если кортеж <a,b> принадлежит AфB.

Важно понять, что бинарное отношение - это всего лишь множество двоек.
>> No.76238 Reply
>>76222
Обычно торическую поверхность называют тором, а
> декартово произведение окружности на круг
полноторием.
>> No.76257 Reply
>>76223
> Важно понять, что бинарное отношение - это всего лишь множество двоек.
А ещё, если считать для любых a, b, c, {a, {a, b}} {a, {a, b}} упорядоченной парой (a, b) с первым членом a и вторым членом b, а ((a, b), c) - упорядоченной тройкой (a, b, c), имея достаточно сильную теорию множеств интерпретировать некоторое явление в ней как натуральные числа, определить индуктивно кортежи длины n для всякого n (для 1 как единственный член, для 0, полагаю, как пустое , но пока мне привычнее начинать натуральный ряд единицей), то получится, что всякое отношение является бинарным.
труисты-труистики.jpg
>> No.76259 Reply
>>76222
Мне кажется, что такое доказательство ("наденем на обруч диск и провернём") обманчиво просто.
Например, что будет декартовым произведением окружности на отрезок? Цилиндр? А почему не лента Мёбиуса? Её тоже можно получить, "надев отрезок на обруч и повернув".

Декартово произведение окружностей S^1 \times S^1 - это enwiki://Clifford_torus
>> No.76262 Reply
Как решать тригонометрические задачки десятого класса школы? Вот открываю я Сканави на восьмом разделе и совершенно не понимаю, как это делать. При этом я могу держать перед собой все формулы, но всё равно ничего не понять.
>> No.76264 Reply
>>76262
Читай учебник Мордковича для десятого класса. Самое начало. Ссылка выше по треду.
>> No.76265 Reply
>>76264
Спасибо, почитаю.
>> No.76266 Reply
>>76259
> Её тоже можно получить, "надев отрезок на обруч и повернув".
Склеивать края этой ленты ты не имеешь права, так как каждой точке окружности должен соответствовать только один отрезок. У тебя получится закрученная поверхность цилиндра.

>>76257
Бурбакист детектед, все в наглядность. Читал книжку Э. Ландау "Основы анализа"? Он там строго конструирует числа вплоть до комплексных на основе аксиом Пеано, очень интересная вещь.
>> No.76267 Reply
>>76132
Я тебя ненавижу.
>> No.76268 Reply
File: IMG018.jpg
Jpg, 24.92 KB, 240×320 - Click the image to expand
edit Find source with google Find source with iqdb
IMG018.jpg
Анон, я сейчас сижу на лекции по матану, первокурсота. Изучаем матрицы, определители, порядки, миноры, алгебраические дополнения, свойства, транспонирование и иже с ним. Лектор говорит тихо и непонятно, пишет коряво, после половины пары заебался писать. Подскажи где всё это можно найти в человеческом виде, с меня нихуя. Лекцию потом сфотографирую и перепишу себе в любом случае.
>> No.76270 Reply
>>76268
Колмогоров, Фомин. "Элементы теории функций и функционального анализа". Из этой книги тебе нужны первые три параграфа про теорию множеств.
http://rghost.ru/40200544

Головина. "Линейная алгебра и некоторые её приложения". Подойдёт, если тебе нужно быстро въехать в тему.
http://rghost.ru/40200394

Ильин, Позняк. "Линейная алгебра и аналитическая геометрия". Традиционный учебник российских университетов. Устарел, но всё же полезен.
http://rghost.ru/40200128


Когда прочтёшь, рекомендую ознакомиться с книжками Кострикина-Манина, Куроша и Ван дер Вардена.
>> No.76275 Reply
>>76266
> Читал книжку Э. Ландау "Основы анализа"? Он там строго конструирует числа вплоть до комплексных на основе аксиом Пеано, очень интересная вещь.
Мало читал, но представление имею. Это я её здесь первым упомянул. Уступает, на мой взгляд, лучшей книге такого уровня и направленности "Числовые системы" Соломона Фефермана. Например, в последней имеется в виду неформальная аксиоматическая теория множеств против неформальной неаксиоматической у Эдмунда Ландау, также в "Основах анализа" нет определения функции внутри теории множеств на манер >>76223.
А вообще, это неверный подход, он приводит к привязанности человека к теории множеств, когда он избегает отдельного изучения других теорий, не понимая, что теория множеств равноправна, к примеру, с теорией натуральных чисел, теорией групп или геометрией.
>> No.76297 Reply
>>76201
Это ты на тифаретнике вопрос задавал, или это новое поколение будущих учоных поголовно такое?
Не бублик, а поверхность бублика, кстати.
Иди-ка ты на гумнитария вообще и не парься.
>> No.76298 Reply
>>76268
> на лекции по матану
> Изучаем матрицы, определители, порядки, миноры, алгебраические дополнения, свойства, транспонирование
//Math in Rushka//
>> No.76299 Reply
>>76268
А найте это можно в человеческом (первокультурном) виде в Курсе Алгебры Э.Б.Винберга.
Гарантирую, что вам даже не объясняют, почему произведение матриц определено именно так, а не иначе.
>> No.76300 Reply
File: 19920927lr.jpg
Jpg, 164.29 KB, 1043×979 - Click the image to expand
edit Find source with google Find source with iqdb
19920927lr.jpg
Как с помощью подбрасывания монетки равновероятно выбрать одно из N чисел?
>> No.76301 Reply
>>76297
Математики ближе к гуманитариям, чем к естественникам, хотя несравненно поверхнестнее. Ну или по меньшей мере равноудалены от тех и других.
>> No.76303 Reply
>>76299
Это ещё что по сравнению с определением матрицы как прямоугольной таблицы.
Я возражаю не потому, что знаю более умное определение и это мне льстит, а потому что отождествляю понятия математической теории и формальной системы, а случай прямоугольной таблицы не вызывает ощущения формальной системы.
>> No.76304 Reply
>>76132
> [батхёрт-комплект]
Ничего себе, как тебе припекло. У тебя был неудачный опыт?
Вообще, перечисленное тобой частично имеет место быть, но за всё же надо платить. В частности, за возможность общения с умными людьми, которые ещё и разделяют твои интересы, приходится платить терпимостью к фрикам, иначе никак.
>> No.76305 Reply
>>76300
Уточню: с вероятностью 1/N
>> No.76306 Reply
>>76303
> по сравнению с определением матрицы как прямоугольной таблицы
Няша, маленькая просьба. Если ты критикуешь некоторое определение, предлагай, пожалуйста, другое, более изящное на твой взгляд. Как бы ты определил матрицу?
>> No.76308 Reply
>>76306
Я бы как массив коэффициентов например. Выраженный в виде таблицы.
>> No.76309 Reply
>>76306
Вектор векторов, не?
>> No.76315 Reply
>>76306
У Бурбаки есть определение. Неужто тебе известно только то, что про таблицы? Не обижайся ни если да, ни если нет. Я действительно задаю вопрос.
>> No.76316 Reply
>>76308
Ты прав. Только
> Выраженный в виде таблицы.
отдельно от определения.
>> No.76317 Reply
File: 1346873020433.png
Png, 1.44 KB, 300×20 - Click the image to expand
edit Find source with google Find source with iqdb
1346873020433.png
>>76309
Была похожая идея, кстати, в своё время. Я тогда определял матрицу как только любую конечную последовательность конечных последовательностей с равными областями определения.
Капча об увольнении по причине сердечных недугов, что ли?
>> No.76318 Reply
>>76316
Поэтому отдельным предложением. Всё тот же первокурсник кстати. Книги понятнее лекций, но в лекции не надо искать суть.
>> No.76319 Reply
File: Бурбаки_Алгебра.JPG
Jpg, 95.28 KB, 748×467 - Click the image to expand
edit Find source with google Find source with iqdb
Бурбаки_Алгебра.JPG
>>76313
У бурбаков в книжке "Алгебра. Часть 1" матрица определяется и вводится именно как таблица, они лишь формализуют понятие таблицы. Ван дер Варден (ссылка на pdf выше по треду, страница 91) при вводе понятия матрицы пользуется действием линейного преобразования на некоторый базис, это, на мой взгляд, практичнее и логичнее.

Когда говоришь о том, что некоторое определение вида "$сущность$ - это ..." маразматично, предлагай, пожалуйста, своё собственное определение, чтобы можно было в будущем сослаться на твои слова, если кто-нибудь спросит, что такое $сущность$.
>> No.76339 Reply
Нам определяли матрицу примерно так:
I,J - мн-во строчных/столбцовых индексов, M - какое-то множество
Тогда отображение m:IxJ -> M будет матрицей.
>> No.76342 Reply
>>76318
> Поэтому отдельным предложением.
Понял. Хорошо, что ты в этом разобрался и формализовал.
>>76319
Не хотел спойлерить материал Бурбаки, чтоб не так сразу. А то когда резко выдают информацию, то теряешься, думаешь, дали бы ссылку и сам бы посмотрел.
Определение на мой взгляд не маразматично, а научно. Оно не математическое.

Да, где-то выше я писал, что математическая логика не область математики. Это происходит из моего отождествления понятий математическая теория и формальная система. Математическая логика - наука и методы её научны. Эта область философии в том числе изучает метатеории формальных систем, в то время как теории математической логики не обязательно оформлены как формальные системы.
Формальная система - понятие математической логики, но давать ей определение, при котором это понятие индуцировано другими, я не вижу смысла. Для науки привычнее описательное определение.
Можно, к примеру, объяснить, что такое язык, аксиомы и правила вывода, составляющие формальную систему.
>> No.76344 Reply
>>76342
Твой подход хорош, если нужно упорядочить уже имеющийся в голове материал. Использовать книжки Бурбаки в качестве учебника и грузить формализмом людей, у которых со словом "матрица" ассоциируется голливудский фильм, неразумно.
>> No.76348 Reply
>>76268
Странно. У нас все, что ты написал, было по линалу.
>> No.76380 Reply
>>76348
Ничуть не странно. Я полагаю, слово матан анон употребил иронично.
>> No.76389 Reply
>>76380
Да, это была лекция по линейной алгебре. Слово матан я употреблял не столько иронично, сколько для всей университетской математики.
>> No.76404 Reply
File: 67886.jpeg
Jpeg, 54.42 KB, 301×301 - Click the image to expand
edit Find source with google Find source with iqdb
67886.jpeg
>>76380
> слово матан анон употребил иронично
курс анализа в российских вузах итак вызывает одну иронию
>> No.76408 Reply
File: perelman-newsmoldova.ru.jpg
Jpg, 99.35 KB, 450×304 - Click the image to expand
edit Find source with google Find source with iqdb
perelman-newsmoldova.ru.jpg
>>76404
Но некоторые умудряются даже этот примитивный курс не понимать.
В забугориях дело обстоит не лучше, если ты учишься не в Гарварде или Оксфорде.
>> No.76409 Reply
>>76408
> В забугориях дело обстоит не лучше, если ты учишься не в Гарварде или Оксфорде.
Диванный ты.
>> No.76422 Reply
>>76409
Нет ты.
http://www2.gre.ac.uk/study/courses/ug/mat/g900 - гринвичский бакалавриат. Линейка на втором году обучения, например.
>> No.76435 Reply
File: 134694549172341.jpg
Jpg, 52.81 KB, 604×403 - Click the image to expand
edit Find source with google Find source with iqdb
134694549172341.jpg
Всем добра. Так вот. Задачка на комбинаторику. Есть 5 равнозначных объектов А, а также 4 равнозначных объекта B. Сколькими способами можно расставить эти объекты, чтобы два объекта B не находились рядом?
>> No.76438 Reply
>>76422
То, что есть такой вот гринвичский, это ВООБЩЕ не говорит о том, что все остальные такие же.
  
Это я даже не затрагиваю лигу плюща и прочие топы среди топов а-ля МИТ, Калтех, Кембридж.
>> No.76440 Reply
>>76435
Общее число комбинаций минус все варианты, включающие вариант ВВ

Мимопроходил
>> No.76445 Reply
File: 1326473908656.png
Png, 337.30 KB, 679×424 - Click the image to expand
edit Find source with google Find source with iqdb
1326473908656.png
Привет математики. У первокурсника уже есть вопросы. Пусть |X|=m, |Y|=n. Как найти число отображений, биективных отображений, инъективных отображений и сюръективных отображений? Объясните пожалуйста доброматематики.
>> No.76446 Reply
>>76438
> ВООБЩЕ не говорит о том, что все остальные такие же.
Говорит. Обычный российский университет сильнее обычного европейского, но "звёздных" университетов, сравнимых с MIT или Гарвардом, у нас нет.
>> No.76451 Reply
>>76446
Нет, дело в том, что я видел программы и не звёздных ВУЗов - и они не были похожи на твой пример (где, судя по всему, скорее математика с уклоном во статистику и вычисления).
>> No.76452 Reply
File: 1302760344_xzibit270.jpg
Jpg, 9.88 KB, 270×198 - Click the image to expand
edit Find source with google Find source with iqdb
1302760344_xzibit270.jpg
>>76446
> Обычный российский университет сильнее обычного европейского
Тебе об этом Садовничий рассказал?
>> No.76457 Reply
File: 873951.jpg
Jpg, 216.97 KB, 960×890 - Click the image to expand
edit Find source with google Find source with iqdb
873951.jpg
>>76435
У тебя есть слово BABABAB, тебе ещё нужно две буквы A впихнуть на какую-нибудь из пяти позиций каждую. Итого 15 вариантов.

>>76445
Сюръективных - n!S(m,n) - это числа Стирлинга второго рода, остальное совсем просто же, на пальцах решается: например, всех отображений - ну надо куда-то отобразить один элемент, другой тоже потом не важно куда, потом третий в какой-нибудь из n имеющихся, ..., потом m-ый. Итого n^m вариантов.
>> No.76459 Reply
>>76445
Отображение A->B - это когда каждому элементу A соответствует некоторый элемент из B.
Сюръекция - когда каждый элемент из B имеет прообраз.
Инъекция - когда разные элементы A отображаются в разные элементы B.
Биекция - инъекция и сюръекция одновременно, взаимно-однозначное отношение.

Имеем множество X, ||X|| = m, и множество Y, ||Y|| = n.
Рассмотрим частный случай, чтобы увидеть закономерность. X = {a,b,c}, Y = {1, 2}.
||X|| = 3, ||Y|| = 2.
Построим всевозможные отображения X->Y.
1) a -> 1, b -> 1, c -> 1
2) a -> 1, b -> 1, c -> 2
3) a -> 1, b -> 2, c -> 1
4) a -> 1, b -> 2, c -> 2
5) a -> 2, b -> 1, c -> 1
6) a -> 2, b -> 1, c -> 2
7) a -> 2, b -> 2, c -> 1
8) a -> 2, b -> 2, c -> 2
Видно, что каждому элементу из X мы можем поставить в соответствие ровно n элементов, то есть отображений будет ровно
n*n*...(m раз)...*n = n^m.

Теперь разберёмся с сюръекциями. Сюръекция - это когда каждый элемент в Y имеет прообраз в X, значит, если в Y будет больше элементов, чем в X, то какие-то элементы из Y никогда не будут иметь прообразов в X, так как каждому лементу X может соответствовать только один элемент из Y. То есть если n>m, то сюръекций ноль. Предположим, что n<=m. Вернёмся к рассмотренному выше частному случаю. Видно, что отображения 1 и 2 не являются сюръекциями. То есть чтобы построить несюръективное отображение, исключаем из Y один какой-нибудь элемент и строим всевозможные отображения X->Y, все они будут несюръективными. Их, как установлено выше, будет (n-1)^m. Замечательно, что в Y мы можем выбрать n элементов для исключения, значит, возможных несюръективных отображений будет равно n раз по ((n-1)^m). Все остальные отображения будут сюръективными. Значит, сюръективных отображений X->Y ровно n^m - n((n-1)^m). В нашем частном случае их будет 2^3 - 2((2-1)^3) = 8-2=6.

Инъекцию и биекцию обдумай сам.
>> No.76460 Reply
File: AfricSimone.jpg
Jpg, 45.97 KB, 393×599 - Click the image to expand
edit Find source with google Find source with iqdb
AfricSimone.jpg
>>76452
Обычный европейский универ похож на Гринвич. В Гринвиче математика преподаётся слабо.
>> No.76462 Reply
>>76408
> Но некоторые умудряются даже этот примитивный курс не понимать.
На мой взгляд, этот курс напротив слишком сложный для математики. Не математически сложный.
> В забугориях дело обстоит не лучше, если ты учишься не в Гарварде или Оксфорде.
И подобного класса, ты, наверное, имел в виду.
>>76446
Но не в математике сильнее. Математики в университете нет.
Да, студенты как бы решают более сложные задачки из книг, но математику не изучают. На самом деле они получают правильные ответы, но не решают задачу, так как понятия, о которых идёт речь, студентам не известны.
Выше, например, обсуждались матрицы. Я не считаю, что способность человека получить правильные ответы к задачам убеждает нас в его математическом образовании, при том что этот человек считает матрицу прямоугольной таблицей. Наоборот, я буду считать, что в данном вопросе математики он не искушён.
Пусть, к примеру, нужно вычислить определитель матрицы над R и правильный ответ к задаче 2. Студент получит этот ответ, но он не знает, что такое матрица, что такое определитель и что такое 2.
Можно было бы даже играя на том, что студенту понятия не известны, составить задачи, которые смотрелись бы безобидно, но ответы выходили неправильные.
>> No.76463 Reply
>>76459
> равно n раз по ((n-1)^m)
Отображения, у которых в образ не попадает несколько элементов, ты считаешь по несколько раз.
>> No.76465 Reply
>>76463
Чёрт.
>> No.76466 Reply
>>76460
> В Гринвиче математика преподаётся слабо.
разумеется, слабо.
Мест, где готовят математиков-исследователей, а не школьных учителей, вообще по всей планете ну никак не больше, скажем, тридцати.
Дело в том, что в РФ математику не преподают вообще, то есть буквально.
>> No.76467 Reply
File: september.jpg
Jpg, 60.72 KB, 320×240 - Click the image to expand
edit Find source with google Find source with iqdb
september.jpg
>>76465
Вот такая атмосфера в треде мне нравится. Рабочая, боевая ...
>> No.76487 Reply
>>76446
> Обычный российский университет сильнее обычного европейского
Да хуйня же.
You mean "звёздный по рашинским меркам университет сильнее обычного европейского".
>> No.76489 Reply
>>76301
> Математики ближе к гуманитариям, чем к естественникам
> хотя несравненно поверхнестнее
"Изучая" курс "высшей математики" в рашкинских недовузах, действительно можно приблизиться к гуманитарию, по уровню интеллекта и квадратно-гнездовому мышлению хотя бы.
>> No.76490 Reply
>>76303
> Это ещё что по сравнению с определением матрицы как прямоугольной таблицы
Оно как раз так и определяется, как можно умнее?
Запомни, что важно не само множество, а структура на нём, то есть свойства операций, морфизмов (в данном случае - свойства сложения и умножения матриц), какая-нибудь метрика, топология или хотя бы отношение порядка.
А множества без структуры не особо отличаются друг от друга. Конкретно, они бывают конечными и бесконечными, счётными и несчётными.
>> No.76491 Reply
>>76319>>76339>>76342
Математика как раз и заключается в том, чтобы стало понятно, что все определения изоморфны и никакой разницы в итоге нет.
По мне, так нормально надо объяснять:
Множества (наивные, основы ZF, рассмотрение парадокса Рассела), отображения - Группы, кольца, поля, операции, морфизмы (вот тут можно осторожно запихнуть категории) - модули над кольцами - векторные пространства (базисы-хуязисы).
Вот здесь то блять и начинается линейная алгебра, и матрицу можно определить по-всякому, всё будет понятно из свойств операций над матрицами.
>> No.76493 Reply
>>76490
> Оно как раз так и определяется, как можно умнее?
Математическое определение основывается на замене выражения языка формальной системы не относящимся к этому языку определяемым выражением. Поэтому в предложенном тобой случае необходимо понятие прямоугольной таблицы, также определяемое либо неопределяемое, что вызывает затруднение. Однако в достаточно сильной теории множеств мы можем рассмотреть семейство с множеством индексов-декартовым произведением конечных множеств (как в определении матрицы Бурбаки), что является удовлетворительным решением задачи определения свойства выражения обозначать матрицу.
> Запомни, что важно не само множество, а структура на нём, то есть свойства операций, морфизмов (в данном случае - свойства сложения и умножения матриц), какая-нибудь метрика, топология или хотя бы отношение порядка.
> А множества без структуры не особо отличаются друг от друга. Конкретно, они бывают конечными и бесконечными, счётными и несчётными.
Не совсем мне ясно, о чём ты пишешь. Если ты имеешь в виду, что всё равно, будет ли это множество прямоугольных таблиц или семейств с множеством индексов, являющимся декартовым произведением конечных множеств, то я замечу, что первое понятие нуждается в дополнительном разъяснении.
>> No.76499 Reply
>>76489
Сколько ты там уже лекций в НМУ посетил? 1? 0? А понту как у комиссарши.
>> No.76503 Reply
Не раз уже читаю заявления в духе "в российских вузах математики нет", и вообще, что ее, по-видимому, преподают только в топовых американских университетах. Я вообще не в теме, но со стороны это кажется абсурдом. Я еще мог бы понять то, что уровень наших вузов ниже. Чему тогда учат под крышами мехматов НГУ, МГУ, ВШЭ, того же НМУ и т.д.? Почему многие студенты и выпускники наших вузов успешно продолжают учебу за границей, до сих пор получают филдсовские премии?
Так троллинг ли это, или есть какие-то объективные предпосылки к подобным высказываниям?
>> No.76508 Reply
>>76503
Смотря что называть преподаванием математики. Исследователей, которые работают на передовых фронтах науки готовят действительно только в аспирантурах топовых американских вузов и парочке европейских. Школьных учителей, обслугу для инженеров, математиков-сисадминов и т.п. готовят везде и российские вузы в этом ничуть не лучше каких-нибудь, скажем, Румынских.
> Почему многие студенты и выпускники наших вузов успешно продолжают учебу за границей
Потому что в РФ ещё остаются (но очень мало) люди, которые разбираются в науке. Даже на мехмате можно таких найти человек пятнадцать. Они читают сильные спецкурсы, пытаются помочь серьёзно настроенным студентам. Дело в том, что в самой учебной программе мехмата (не говоря уже о провинциальных заведениях, где всё на порядок хуже) нет никакой математики, которая появилась после 30х.
> ВШЭ
Выглядит, конечно, многообещающе, но в этом году был только первый выпуск бакалавриата (причём хуёвый, в 2008 брали всех подряд).
> НМУ
Постепенно дряхлеет из-за оттока людей и, соответственно, спецкурсов, на Матфак.
В НМУ ещё можно научиться математике, но как раз "основное" место учёбы будет этому мешать в первую очередь.
> получают филдсовские премии
Три человека после развала СССР.
Перельман -- прямой наследник школы А.Д.Александрова, остатки которой и ныне еле-еле живы в Питере.
Окуньков -- закончил НМУ.
Смирнов -- учился как раз в топовой американской аспирантуре (Caltech).
>> No.76513 Reply
>>76508
> парочке европейских
А в каких именно?
>> No.76524 Reply
>>76508
> Три человека после развала СССР
Она и вручалась-то всего 5 раз после развала СССР.
И каждый раз дают втч кому-нибудь из б. СССР или России.
Другое дело, что все, кроме Перельмана, давно уехали. Их русскими можно считать уже с натяжкой.
>> No.76530 Reply
File: am-Wayne_Barlowe_Filter.jpg
Jpg, 792.46 KB, 1076×1426 - Click the image to expand
edit Find source with google Find source with iqdb
am-Wayne_Barlowe_Filter.jpg
Посоветуйте диванному математику хороший список литературы (с примерами и задачками) по теории вероятности, математической статистике, случайным процессам и всяческим увлекательным дополнениям по этому делу. Спасибо.
>> No.76543 Reply
File: Anatolyev-Interme...
Pdf, 1.03 KB, 595×842
Anatolyev-Intermediate-and-Advanced-Econometrics-P.pdf
File: Анатольев-Курс-ле...
Pdf, 0.70 KB, 595×842
Анатольев-Курс-лекций-по-эконометрике-для-подготов.pdf
File: Анатольев-Курс-ле...
Pdf, 0.57 KB, 595×842
Анатольев-Курс-лекций-по-эконометрике-для-продолжа.pdf

>>76530

Тервер: Ширяев - Вероятность-1 (есть в мцнмо 210 р. можно купить бумажный, пригодится)
Статистика: мне нравится Casella, Berger - Statistical Inference
Байесовская статистика: Bernardo, Smith - Bayesian Theory
Предельные теоремы: Davidson - Stochastic Limit Theory

Оче рекомендую лекции Анатольева по эконометрике (с задачами, приложил к посту)
Hayashi - Econometrics

Далее, если заинтересует какая-то конкретная тема, можно читать

Hamilton - Time Series Analysis
Lütkepohl - New Introduction to Multiple Time Series Analysis

Horowitz - Semiparametric and Nonparametric Methods in Econometrics
Обзорная статья Heiler - A Survey on Nonparametric Time Series Analysis

Efron, Tibshirani - An Introduction to Bootstrap
Hall - The Bootstrap and Edgeworth Expansion
Smeekes - Bootstrapping Nonstationary Time Series

Hall - Generalized Method of Moments

Журнал http://quantile.ru/, там бывают хорошие обзоры
Есть многотомная энциклопедия Handbook of Econometrics. По темам, есть теория и приложения
>> No.76544 Reply
>>76530
Да, в Анатольеве сначала идёт "эконометрика для продолжающих" (intermediate), а потом для "подготовленных" (advanced)
>> No.76545 Reply
>>76543
Спасибо.
>> No.76554 Reply
File: 165453_pic1.jpeg
Jpeg, 21.94 KB, 280×337 - Click the image to expand
edit Find source with google Find source with iqdb
165453_pic1.jpeg
>>76446
Я закончил магистратуру в немецком вузе (называть не буду, ибо деанон, но вуз престижный), однако решил вернуться в Россию, поступил в родную питерскую аспирантуру, к старому научруку. Считаю, что тут достиг бы в своих исследованиях больше, чем за два года там, да и в целом был бы состоятельнее.

Меня вообще удивляют порой антипатриотические настроения среди якобы "илиты" матмеховского бомонда. Все будущие бачелоры и спецы в один голос вопят, что "мы съебем хоть куда, только бы здесь не оставаться, здесь говно тупо плохо, науки нет, везде совок, математика застряла в 60-ых, кудах-тах-тах и тд". Я побыв там могу смело заявить: средний бакалавр питерского матмеха сильнее среднего бакалавра любого из немецких вузов.

Так что и у нас можно вполне себе состояться, если есть голова на плечах. Благо пока всё бесплатно, а толковые учёные пока ещё есть (хотя отток идёт сильнейший, скоро их не будет).
>>76462
> Я не считаю, что способность человека получить правильные ответы к задачам убеждает нас в его математическом образовании
"Задачи" разные бывают. Та же теорема Пуанкаре - вполне себе задачка, но у тебя ведь нет сомнений в образованности Перельмана?
Есть задачи, грубо говоря, вычислительные, а есть, как их называл один наш лектор, "творческие". Математическая образованность заключается в умении решать и те и другие, а это возможно только при понимании теории и понимании, как её применять на практике.
>>76466
> Дело в том, что в РФ математику не преподают вообще, то есть буквально.
Да-да, а ещё у нас медведи в шапках-ушанках по улицам в минус 40 ходят и играют на балалайках.
>>76503
Скорее всего это зависть. Многим (да почти всем, сужу по своему выпуску) родители и сокурсники пытаются впарить две вещи:
a) Науки здесь нет, штаны не на что будет купить. Не иди в науку.
b) Ну если уж решил продолжать образование, пробивайся куда угодно, только не здесь.
Но есть одно но : чтобы поступить куда-нибудь зарубеж, нужны мозги и деньги. При этом на "мозги" требования вполне себе нормальные, ну, скажем, средний балл около 4.5 - это вполне себе реально иметь (это не критерий, это я навскидку говорю, чисто чтобы представление вы имели). А вот денюжки нужны хорошие, которых у большинства нет. Вот и сидят озлобленные бакалавры, работают флэш-программистами и хают наши университеты (я таких троих с бывшего выпуска знаю).
Я не пытаюсь обобщать, я лишь на основе своих наблюдений высказываю гипотезу. Мне это всё видится по крайней мере именно так.
Кстати, дико жалею, что не могу ходить в НМУ (только тифаретные лекции/курсы читал).
>>76508
Вот, хоть один человек итт "в теме".
>> No.76559 Reply
Привет, анон. Вот я и посетил первую лекцию НМУ. То, что мне было скучно, можно и не говорить, первая лекция была вводной. Но я попробовал представить себе, что не знаю о математике ничего, и получать удовольствие. Так вот, это был пиздец.
Лектор бормотал себе под нос, дикая нестрогость, непоясняемые фразы из неизвестного материала. Например, в начале преподаватель объяснял, как натуральные числа постепенно расширяются до действительных, когда коснулся рациональных, то было "так, ну, некоторые такие дроби определяют одно и то же число, поэтому нам надо факторизовать это множество по какому-то отношению эквивалентности...". И вот таких мелочей уйма, особенно поразило использование "стремления к нулю" до введения понятия предела. Я-то его понимал, а вот большинство, думаю, нет.
Это всегда так будет?
>> No.76560 Reply
>>76559
> Это всегда так будет?
Ну преподаватель преподавателю рознь. Некоторые очень хорошо читают.
Ты можешь скачать какие-нибудь другие лекции и оценить http://ium.mccme.ru/IUM-video.html
>> No.76561 Reply
>>76559
а чего ты хотел от лектора, который ставит Фихтенгольца в список литературы?
Первокурсникам этого года зато очень повезло с геометрией.
>> No.76562 Reply
>>76554
> Я закончил магистратуру в немецком вузе
> Я побыв там могу смело заявить
> магистратуру
> в немецком вузе
Ты бы ещё польский детский сад закончил и что-то заявлял.
Речь идёт об аспирантуре, phd. И факт в том, что выпускник мехматовской или что-то в этом духе аспирантуры, который не надрывался дополнительно в НМУ или ПОМИ и которому не повезло со звёздным научруком, в науке инвалид, не в состоянии он уже заниматься наукой, время потеряно.
>> No.76563 Reply
>>76559
Это плохие новости для меня. Я хотел посмотреть видеозаписи лекций Сабира Меджидовича Гусейн-Зада, но он часто сбивался, говорил нечётко, писал на доске неразборчиво и уродовал слова уменьшительно-ласкательными суффиксами и т.д. Поэтому я решил что толку от его лекций для меня мало. И тут снова такое безобразие. Кстати, Раскин мне тоже не понравился.

Я не хочу читать Фихтенгольца и у меня нет желания читать Зорича. Я могу читать Львовского, но его книга написана сжато и плохо. Мне нужно что-то ещё к его лекциям. Я нашёл две книги, Real Mathematical Analysis и Mathematical Analysis: A Concise Introduction. Какая из них больше подходит для первого курса НМУ?

Интересно, а топология на втором курсе будет?
>> No.76564 Reply
>>76559
Привет.
Лекции не видел, потому как соотнёс количество народа-кислорода и свой интерес к читаемой теме, и пошёл на лекцию по анализу 2-го курса, где прекрасный Гусейн-Заде угорал по разным буковкам , пока они не закончились.
Взял листок первого курса, там какой-то фихтенгольц на фихтенгольце фихтенгольцем погоняет.
> Например, в начале преподаватель объяснял, как натуральные числа постепенно расширяются до действительных, когда коснулся рациональных, то было "так, ну, некоторые такие дроби определяют одно и то же число, поэтому нам надо факторизовать это множество по какому-то отношению эквивалентности..."
Это лучше, чем я думал. А как он делал R, через дедекиндовы сечения, или как множество классов эквивалентности фундаментальных последовательностей из Q? Лучше анализ начинать, мне кажется, с основ топологии (первые две главы Вербицкого).
Вообще, лучше читай книги (хорошие), лекции по анализу понятными и хорошими способен сделать далеко не каждый.
Короче, на лекции по анализу 1-го семестра я ходить не буду, я всё это уже слышал, надеюсь, дальше будет лучше. Листки сдавать буду и всё.
Хотя мне анализ не симпатичен в любом из видов, в которых я его встречал.
> поразило использование "стремления к нулю" до введения понятия предела
Для того, чтобы так говорить, достаточно определить понятие фундаментальных последовательностей, я думаю, он это сделал.
>> No.76565 Reply
>>76563
Топология будет во втором семестре.
>> No.76566 Reply
>>76563
> Интересно, а топология на втором курсе будет?
Скорее всего, на 1-м курсе весной.
>> No.76567 Reply
>>76564
Надо было с тобой идти.
> > А как он делал R, через дедекиндовы сечения, или как множество классов эквивалентности фундаментальных последовательностей из Q?
Дедекиндовы сечения, к моему ужасу.
Да знаком мне этот материал, просто вдруг будет что-нибудь внезапно новое. Да и люди могут попасться интересные.
> > Для того, чтобы так говорить, достаточно определить понятие фундаментальных последовательностей, я думаю, он это сделал.
Не-а, он этого не сделал. Ты слишком высокого мнения о нем.
>> No.76568 Reply
>>76567
> Дедекиндовы сечения, к моему ужасу.
Ну это не ужасно, я так спросил, интересно было.
Оно не хуже, в принципе, просто обычно стараются делать НЕ ТАК, как в Фихтенгольце, потому как Фихтенгольца все в МГУ наслушались уже.
> Не-а, он этого не сделал.
Странно, а он про последовательности говорил вообще? Если что, на R^1 "фундаментальная последовательность" = "последовательность Коши" = "последовательность, имеющая конечный предел" (определять понятие "предел" там не требуется, оно, наоборот, оттуда возникает).
>> No.76569 Reply
>>76568
Сначала было все про введение действительных чисел (лемма о вложенных отрезках доказывалась, как альтернатива аксиоме полноты), а потом уже было все про последовательности.
>> No.76570 Reply
>>76568
selffix: не на R^1, а вообще.
Конкретно, каждое число из R бесконечно приближаемо числами из Q.
>> No.76571 Reply
>>76569
Ну ладно.
Прости его, он молодой и, судя по всему, с мехмата.
Не относись так серьёзно к анализу.
Алгебра, вроде, будет адекватная.
Ну и геометрия, говорят, хорошая.
>> No.76573 Reply
>>76564
> пошёл на лекцию по анализу 2-го курса
Не знаешь, видео записывали, или нет?
>> No.76574 Reply
>>76573
Камера на штативе не стояла, вроде, зато вверху висела какая-то, но это не та, по-моему, а для наблюдения.
У него лекции нормальные, вроде, просто нужно предварительно почитать, а то в буковках запутаешься, он их, по-моему, очень любит, а пишет на доске жутко, не то, что Сосинский (у него очень красиво получается писать на доске).
Хотя это моё мнение, я только один раз был (сегодня).
В общем, не сильно полезно его записывать.
Вот Сосинского - полезно, у него странные программа и лекции, но интересные.
>> No.76578 Reply
Анон, а у рациональных чисел существуют какие-нибудь расширения, помимо действительных чисел?
>> No.76582 Reply
>>76574
А можешь листок с задачами отсканировать и выложить? Интересно.
>> No.76583 Reply
>>76554
> "Задачи" разные бывают. Та же теорема Пуанкаре - вполне себе задачка, но у тебя ведь нет сомнений в образованности Перельмана?
Я читал его интервью, и если оно соответствует действительности, то я не считаю Перельмана коллегой в математике.
>>76564
> Для того, чтобы так говорить, достаточно определить понятие фундаментальных последовательностей, я думаю, он это сделал.
Я подумал, что понятие бесконечно малых последовательностей. Ты это хотел сказать?
>> No.76584 Reply
>>76578
Рациональные числа, очевидно же. смайл.gif Также рациональные числа с несобственными числами, действительные числа с несобственными числами, комплексные числа.
>> No.76585 Reply
File: 0307245v1.pdf
Pdf, 0.12 KB, 612×792
0307245v1.pdf
File: 0211159v1.pdf
Pdf, 0.33 KB, 612×792
0211159v1.pdf
File: 0303109v1.pdf
Pdf, 0.26 KB, 612×792
0303109v1.pdf

>>76579
> и если оно соответствует действительности
Луркоморье утверждает, что это интервью целиком и полностью сгенерировано журналистами КП и что Перельман его не давал. Лурка обычно сведуща в таких вещах.
> я не считаю Перельмана коллегой по математике
Смело. Возможно, есть смысл не читать жёлтую прессу, а вместо этого изучить непосредственно работы Перельмана и уже по их качеству оценивать математический скилл Григория? Приложены.
>> No.76586 Reply
>>76584
Несобственные числа? А что это?
>> No.76587 Reply
>>76586
Бесконечности.
>> No.76606 Reply
Где можно посмотреть, как вводят разные операции над вещественными числами (возведение в степень, логарифм и т.д.)? А то я примерно представляю все это, но есть некоторые пробелы в знаниях.
>> No.76607 Reply
>>76578
> у рациональных чисел существуют какие-нибудь расширения, помимо действительных чисел?
Сколько угодно.
Процесс получение действительных из рациональных называется "пополнение по норме".
"Пополнение" означает, что рассматриваются классы эквивалентности фундаментальных последовательностей и каждый такой класc объявляется новым элементом.
см. тут enwiki://Completeness_(topology)
Подробно этот процесс описан в лекциях преп. Мордехая Вербицмана,
см. тут verbit.ru/MATH/UCHEBNIK/top-book.pdf
Если выбрать p-адическую норму норму, получатся p-алические числа.
см. тут ruwiki://P-адическое_число

Кроме p-адических ещё существует сколько угодно промежуточных конечных расширений между рациональными и действительными, они получаются присоединением корней многочленов.
см. тут enwiki://Algebraic_number_field
такие дела
>> No.76610 Reply
>>76607
А существуют ли иные варианты дополнения рациональных до действительных? В том смысле, что можно ли плотно заполнить прямую, но не при помощи действительных чисел?
>> No.76611 Reply
>>76610
Не понимать. Прямая - это и есть действительные числа.
>> No.76612 Reply
>>76611
Окей, дополнить рациональные числа так, чтобы они удовлетворяли аксиоме полноты, но не при помощи действительных чисел.
>> No.76613 Reply
>>76610
> плотно заполнить прямую
Вообще, "прямой" называется одномерное векторное пространство, которое как группа всегда изоморфно аддитивной группе поля, над которым векторное пространство, слово "прямая" без контекста не имеет смысла.
Если ты имел ввиду другие (эквивалентные, естественно) определения действительных чисел, то см. тут enwiki://Construction_of_the_real_numbers
Но пополнение метрического (или в данном случае нормированного) пространства это наиболее общая и фундаментальная конструкция.
>> No.76614 Reply
>>76612
а, понял сам на первом курсе НМУ испытывал сложности с этой темой
> чтобы они удовлетворяли аксиоме полноты
1. "Полнота" определена для метрических пространств и определяется метрикой. Одно и то же множество в зависимости от метрики может быть как полным, так и не полным.
Например, любое множество с дискретной метрикой является, очевидно, полным метрическим пространством.
1.1 Нетрудно доказать теорему, что любое метрическое пространство может быть изометрически погружено в полное пространство, причём это полное пр-во определено однозначно с точностью до изометрии.
2. Рациональные числа (и вообще кольца) естественно пополнять не просто по какой-то метрике, а по метрике порождённой некоторой нормой.
2.1 Дальше тебе нужно знать, что такое "эквивалентность норм"
см тут ruwiki://Норма_(математика)#.D0.AD.D0.BA.D0.B2.D0.B8.D0.B2.D0.B0.D0.BB[...]D0.BC
Так вот, любое нормирование рациональных чисел эквивалентно или обычному "абсолютному значению" или какой-то р-адичесекой норме, поэтому как не пополняй рациональные числа неизбежно получатся либо действительные, ли какие-то р-адичсекие.
такие дела
>> No.76615 Reply
>>76612
Ой, я дал тебе ссылку на русскую вики, а там эквивалентность норм опрелена только для векторных пространств.
Я боюсь, что фраза "В конечномерном пространстве все нормы эквивалентны" может тебя запутать. Это верно для норм векторных пространств над фиксированным полем. Но сами поля можно нормировать разными способами.
Лучше иметь ввиду следующее простое определение:
Две нормы на поле называются эквивалентными, если любая последовательность, сходящаяся к 0 в одной из норм, сходится к 0 и в другой.
Эквивалентные нормы дают одинаковые пополнения.
>> No.76624 Reply
>>76582
А я не взял.
Он сказал, выложит, но не раньше понедельника.
>> No.76627 Reply
>>76615
Асло у этой теоремы есть персональное название
enwiki://Ostrowski's_theorem
>> No.76675 Reply
Кто ходил в НМУ, по матану для листочков дедлайны есть? И еще, там трансляция в столовую была?
>> No.76678 Reply
>>76675
Трансляция в столовую была, но туда тоже не все поместились.
Обычно дедлайнов нет, однако следует понимать, что неудобно при большом количестве людей сдавать кучу листков за раз.
В крайнем случае, сдаётся всё вместе в конце семестра.
>> No.76697 Reply
>>76678
В прошлом году на анализе нужно было сразу задачи сдавать, по крайней мере в начале семестра.
>> No.76703 Reply
File: 2.jpg
Jpg, 25.76 KB, 500×491 - Click the image to expand
edit Find source with google Find source with iqdb
2.jpg
заменой плоскостей проекций определить натуральную величину двугранного угла abcd.
>> No.76704 Reply
>>76703
"Определить! Быстрабля!" Хоть бы поздоровался.
Няша, просто внимательно прочитай учебник в том месте, где написано про двугранные углы.
>> No.76705 Reply
>>76704
поздно уже читать.
>> No.76706 Reply
>>76705
В будущем тебе всё равно понадобится.
>> No.76708 Reply
>>76697
Про Гусейна я слышал, да.
И в этом семестре - тоже.
Но так больше никто не делает.
Да и листки у него несложные, вроде, особенно если предварительно почитать материал лекции.
>> No.76709 Reply
>>76706
в очень отдаленном если.
>> No.76711 Reply
>>76706
а в ближайшем если я эту хрень принесу решенной мне наконец-то поставят 3 и все от меня отебуться.
>> No.76790 Reply
File: das.png
Png, 6.32 KB, 337×215 - Click the image to expand
edit Find source with google Find source with iqdb
das.png
Всем вечерок, начали изучать теорию множеств и сразу легкий ступор.
Ход решений правильный? Пока с циферками тренируюсь, так легче.
>> No.76822 Reply
>>76790
Нихуя не понел.
Откуда циферки взялись?
И что такое AB?
>> No.76824 Reply
>>76822
Циферки взялись потому что анон решил прочуствовать формулу "на примере", а что такое AB я тоже не понял.
>> No.76833 Reply
>>76822
> что такое AB
Похоже на пересечение. Если это оно, то тогда левую часть нужно довести до конца:
{1,2,3,4} ⋂ {5,6} ∪ {3,4} = ∅ ∪ {3,4} = {3,4}
>> No.76837 Reply
>>74216
Няши, мне нужен ваш совет. Можно ли при условии, что смутно помнишь школьную программу, познавать матанализ по Фихтенгольцу? Не слишком сложный учебник? Заранее спасибо представителям лучшей кафедры.
>> No.76839 Reply
>>76837
Можно. Хотя фихтенгольца тут не любят
>> No.76842 Reply
>>76837
Можно. Примерно на такой уровень знаний он и рассчитан. После него прочитай Зорича, эта книжка посовременнее и немного посложнее. Будет что-то непонятно - спрашивай.

>>76839
> фихтенгольца тут не любят
Причём за дело. Фихтенгольц содрал свой учебник с книжки Коши, а выдал за свой собственный труд.
http://ilib.mccme.ru/djvu/klassik/analysis/koshi.djvu
>> No.76851 Reply
>>76790
Какой же маразм я вчера написал, видать неадекватный был.

>>76822
>>76824
>>76833
Сейчас все перепишу красивенько :3
>> No.76852 Reply
>>76842
> После него прочитай Зорича, эта книжка посовременнее
Ты что серьёзно? Вот так вот от корки до корки читать Фихтенгольца, а потом ещё и два тома Зорича? Да это ж охуеть можно.
>> No.76853 Reply
>>76837
Фихтенгольцевская книжка хороша для многих вещей,
но только не для изучения матанализа.
Эта книга умудрилась устареть до того, как была написана.
>> No.76860 Reply
Анонимус, решай НМУшные листки по геометрии, если ты их ещё не решил, и шагом марш завтра на геометрию.
>> No.76863 Reply
А что, на алгебру никто не ходил, что ли?
>> No.76866 Reply
>>76863
Я ходил. Причем пришел только на семинар, а попал все равно на лекцию, лол.
>> No.76869 Reply
Анон, подскажи, пожалуйста, годный учебник по высшей алгебре, где доступно объясняют полиномы. Тот пиздец, который нам дали на лекции, я так и не смог осилить.
>> No.76870 Reply
>>76869
В любом учебнике по алгебре касаются темы полиномов, но не думаю, что это будет сильно отличаться от того, что вам дали на лекции. Открой лучше какую-нибудь попсу, типа Ван дер Вардена, и если что непонятно, спрашивай здесь.
>> No.76871 Reply
>>76870
Ну у меня есть учебники Куроша и Фаддеева. У Куроша я полиномов так и не нашел, а у Фаддеева сразу наткнулся на что-то типа "ассоциативное коммутативное кольцо с единицей" и сразу закрыл. Сейчас посмотрю Ван дер Вардена, спасибо.
>> No.76872 Reply
>>76871
Так, посмотрел. Где там вообще полиномы? В оглавлении и в указателе ничего.
>> No.76873 Reply
>>76869
Винберг, блять.
"Курс алгебры", блять.
Заебали уже.
>> No.76874 Reply
>>76871
> ассоциативное коммутативное кольцо с единицей
Как будто что-то плохое.
>> No.76875 Reply
>>76871
> наткнулся на что-то типа "ассоциативное коммутативное кольцо с единицей" и сразу закрыл
А что не так то?
>> No.76876 Reply
>>76873
Там я что-то тоже не нашел полиномов ни в оглавлении, ни в указателе , блять
>>76874
>>76875
Не так то, что мы это еще не проходили.
>> No.76878 Reply
Все, википедия прояснила ситуацию. Ебучий преподаватель не объяснил в самом начале, что полином - это обычный многочлен, и я строил какие-то теории насчет стандартного представления числа с системой счисления x. Дальше я как-нибудь сам.
>> No.76879 Reply
>>76876
Поехавший.жпг
> Глава 3
> Начала алгебры многочленов.
Ради тебя прямо откопал из горы книг своего бумажного Винберга и нашёл.
>> No.76880 Reply
>>76876
Тогда знания того что это выражение вида a0 * x^0 + a1 * x^1 + ... + an + x^n.
Но когда изложение ведеться на кольцом его свойства становятся гораздо естественнее и понятнее.
>> No.76881 Reply
>>76878
> не объяснил в самом начале, что полином - это обычный многочлен
пиздец кого в НМУ порой заносит, диву даюсь.
Ты книги читать пробовал, няша?
>> No.76888 Reply
File: hohmodrom_poklon_00.png
Png, 91.12 KB, 434×439 - Click the image to expand
edit Find source with google Find source with iqdb
hohmodrom_poklon_00.png
Доброго вам вечера, достопочтенные мужи. Прибыл я из соседнего уезда недавно отмерший нульчан, но не спешите гнать меня тряпками ссаными, ибо не со своим уставом в чужой монастырь. Хотя насмехаться и унижать можете в полной мере. Прибыл я по делу, да по особо важному. Пилю холодненочку, кратенькую:
21 лвл, программист старший, образование ПТУ, но умения мои мужи по ремеслу оценивают высоко, ибо сам познал сие ремесло с ранних летов. Митол группы не имею. Всё бы ничего, суть ремесла есмь в умении находить да пользовать нужные инструменты, да нестандартно мыслить, но всеж я в отчаянии. Ибо хочу заниматься Computer Science а не лепкой крудв, но институт в соседней губернии мне не светит, стар я годами, да и позабыл многое. Математику да алгебру люблю со школы, но всё успешно позабыто единственное что помню - квадратные уравнения 9кл. Надумал я обучаться науке сей самъ, а дальше будь что будет, постараюсь своими силушками поступить хотя бы на заочное. Ищу совета я. Как быть мне, как учиться науке сей, да что учить чтоб не испытывал я трудностей в этом деле нелегком и зваться гордо программистом мог, да помогать другим мужам позновать компьютер саенс? Челом бью, да мёд вам даю.
:3
Алсо, тред весь не осилил, а из осиленного понял лишь, что ничего толком не понял, простите коли не любим, юродив и гоним, да обратиться больше не к кому
Доброчан же
>> No.76890 Reply
>>76888
> чтоб не испытывал я трудностей в этом деле нелегком
Вот тут я сфейлил. Я имел ввиду трудности с выбором материала. Ибо я знаю только баззворды МАТЕМАТИКА и АЛГЕБРА, а ДИСКРЕТНАЯ БАЛИОЫЬМОИ звучит для меня так же как и пишется.
>> No.76894 Reply
>>76888
Так тебе что нужно?
CS, подготовка к ЕГЭ для поступления или Математика?
Вещи малопересекающиеся.
>> No.76895 Reply
>>76876
> мы это еще не проходили.
Но написано же:
> Программа курса:
> 1. Группы, кольца, поля: определения, примеры, свойства.
Чем вы занимались на первой лекции в итоге?
>> No.76896 Reply
Аноны, кто идёт в четверг на лекцию некоего Миши Вербицкого про гиперболические группы?
>> No.76897 Reply
>>76896
Я иду.
>> No.76903 Reply
>>76894
Малопересекающиеся? Разрыв шаблона. Мне изначально нужно вспомнить основы, дабы продолжать дальнейшее изучение. Приоритетнее, я думаю, подготовка к ЕГЭ, наверное. А в дальнейшем изучении ТОЙ математики, что требуется программисту чтобы угорать по компьютер саенсу, раз она у вас вся разная, эта ваша математика. А в дальнейшем, я думаю, мне уже самому будет ясно что изучать, что не изучать. Ну а если нет, то доброчан же :3 А вообще мне понравилось начало треда, где один доброанон предложил составить программу из трех этапов. Жаль, что тред пока еще не дочитал, может коллективный разум таки что-то и вывел.
>> No.76904 Reply
>>76903
> что требуется программисту
Для начала:
Верещагин, Шень: Начала теории множеств
Верещагин, Шень: Языки и исчисления
Верещагин, Шень: Вычислимые функции
Это тоненькие книжки для ознакомления с основами.
Скачать всё можно тут http://www.mccme.ru/free-books/

Алсо, если хочешь иметь хоть какую-то математическую культуру, то почитай
Курант, Роббинс: Что такое математика
Скачать тут http://ilib.mccme.ru/

Кстати, мне вот интересно, в треде регулярно появляются люди "ничего не знаю, что читать", получают ссылку на фихтенгольца литературу, а взялся ли кто-нибудь из них за эту литературу?
>> No.76905 Reply
>>76904
Спасибо, мил человек. А по поводу САМЫХ основ, не? Или школьная программа не нужна? Алсо, я могу периодически отписываться здесь о результатах :3
>> No.76907 Reply
>>76904
Алсо, Фихтенгольца скачал давно, но почему-то сомневался в надобности данного чтива для меня и решил перечитать войну и мир. Что еще посоветуешь, мил человек? Я сейчас сразу в киндол всё себе солью и буду тихими вечерами почитывать да практиковаться. Ну и поддоброчну еще за фундаментальное чтиво таки.
>> No.76908 Reply
>>76852
> Вот так вот от корки до корки читать Фихтенгольца, а потом ещё и два тома Зорича? Да это ж охуеть можно.
Москва не сразу строилась, доброняша. Прочесть книжку "Математика для чайников" и стать математиком за пару вечеров нельзя. На достойное освоение анализа у тебя уйдёт несколько месяцев кропотливой работы, такие дела. После Зорича хорошо бы проглядеть курс лекций Львовского. http://rghost.ru/40316446

>>76903
> где один доброанон предложил составить программу из трех этапов
Всё ещё составляю. Покопался в программах ЕГЭ, понял, что толковых книжек нет, пишу свою.
Лучшие из ЕГЭшных методичек, на которые я натыкался в процессе гугления - http://rghost.ru/40316374 (подходы к решению задач части C).

Порекомендую тебе как программисту прочитать следующие книжки:
Новиков. "Дискретная математика" http://rghost.ru/40316388
Кристофидес. "Теория графов. Алгоритмический подход" http://rghost.ru/40316389
Ахо, Хопкрофт. "Построение и анализ вычислительных алгоритмов" http://rghost.ru/40316404

Матанализ и линейная алгебра нужны всем серьёзным программистам. Умножение матриц пригодится при работе с директиксом, знание матана на уровне Зорича нужно, чтобы понять вейвлеты.
>> No.76909 Reply
>>76908
Няша, огромное спасибо за книги, схоронил. Но есть одно "НО". Я не уверен, что пойму всё, что должен понять изучая этот материал. И, извините за то, что повторяюсь, спрошу еще раз о каком-нибудь фундаментальном чтиве, дабы вспомнить основы (хотя бы школьную программу, например).
>> No.76910 Reply
>>76909
> Я не уверен, что пойму всё, что должен понять изучая этот материал.
Будто что-то плохое. Когда понимаешь вообще всё - это значит, что ты ничего нового не узнаёшь. Когда тебе будет что-то непонятно, не стесняйся спрашивать.
> хотя бы школьную программу
>>74774
>> No.76911 Reply
>>76910
Спасибо за наводку. Посмотрел другие книги Мордковича, там есть так же учебники 7,8,9 классов. Замечательно, полистаю их прежде чем окунуться в этот увлекательный мир математики. Времени много. Спасибо за помощь, няши :3
>> No.76913 Reply
>>76911
Пожалуйста. Повторяю, не стесняйся спрашивать и пользоваться википедией. В книжках будет много непонятных терминов. Вот у Новикова, например, на первых страницах есть несколько вумных слов, смысл которых он не поясняет.

"Предикат P(x)" - это утверждение, которое может быть истинным или ложным в зависимости от переменного x. Например, предикат НЕГР: НЕГР(Обама) - истина, НЕГР(Путин) - ложь. Или предикат ДЕЛИТСЯНАДВА: ДЕЛИТСЯНАДВА(4) - истина, ДЕЛИТСЯНАДВА(7) - ложь.

"Натуральные числа" - это числа вида 1,2,3,... Для них справедливы аксиомы Пеано.
ruwiki://Аксиомы_Пеано

"Доказательство по индукции". Пусть мы имеем предикат P(n), где n - натуральное число. Хотим доказать, что он верен для всех натуральных чисел. Для этого мы показываем, что предикат P(1) истинен, а затем доказываем, что если предикат P(n-1) верен, то и предикат P(n) верен. Таким образом получаем, что предикат верен для всех натуральных чисел, так как верен для 1, следовательно, верен для 2, следовательно, верен для 3 и так далее.
> Времени много
Это только так кажется. :3
>> No.76914 Reply
>>76904
> а взялся ли кто-нибудь из них за эту литературу?
Мне как-то посоветовали того же "Верещагин, Шень" начал читать про вычислимые функции, очень понравилось, но в связи с делами отложил, планирую завтра продолжить.
>>76905
Для этих книжек не нужно знать практически ничего кроме небольшого числа обозначений.
>> No.76918 Reply
>>76913
> Повторяю, не стесняйся спрашивать и пользоваться википедией.
Википедией в основном и пользуюсь :3
> Это только так кажется
Отчего же, его достаточно. Я не первый раз наступаю на ОЛОЛО ВЫУЧИТЬ $LANGNAME ЗА 24 МИНУТЫ-грабли. Опыт таки подсказывает, что лучше неспешно изучать, вдумчиво читать и больше практиковаться. Спешить некуда.
>>76914
> Для этих книжек не нужно знать практически ничего кроме небольшого числа обозначений.
Вот и замечательно. Но таки для общего развития полистаю школьные учебники.
>> No.76919 Reply
>>76904
> а взялся ли кто-нибудь из них за эту литературу?
Я скачал, начинаю употреблять.
>> No.76926 Reply
Матемачик дорогой, привет!
Никак не могу понять, откуда в аксиоматической теории множеств следует существование таких вещей, как, допустим, объединение счетного числа множеств. Поможешь?
>> No.76928 Reply
Привет, матач. Прошу объяснить как выполнять численное дифференцирование с помощью метода неопределенных коэффициентов.
Можно просто на примере какого-нибудь двучлена.
>> No.76929 Reply
>>76926
Аксиома пары и аксиома полной индукции, не?
>> No.76946 Reply
File: 2012-09-12-13.30.09.jpg
Jpg, 971.15 KB, 2592×1944 - Click the image to expand
edit Find source with google Find source with iqdb
2012-09-12-13.30.09.jpg
Доброаноны, смотрите какая фигня. Мне нужно найти ответы на билеты на пикрелейтеде. Есть ли какие-нибудь учебные порталы, где все это внятно написано. Википедия слишком неразговорчива, гугл лишь запутывает меня, а больше я ничего и не знаю.
>> No.76948 Reply
Ребятки, это снова программист-кун. Запасаюсь литературой. Чт о посоветуете по матанализу? Знакомые советуют кто Фихтенгольца, кто Зоорича, крича что первый устарел. Хотя я не очень понимаю как это может устареть.
>> No.76956 Reply
>>76948
Рекомендую Рудина.
>> No.76957 Reply
>>76926
А индукцию мы можем включать до того, как построены числа?
>> No.76969 Reply
File: godel_photo.jpg
Jpg, 22.90 KB, 297×400 - Click the image to expand
edit Find source with google Find source with iqdb
godel_photo.jpg
>>76948
Ты принципиально не читаешь тред, да?
>> No.76975 Reply
File: aksioma-objedinenija.jpg
Jpg, 27.88 KB, 871×117 - Click the image to expand
edit Find source with google Find source with iqdb
aksioma-objedinenija.jpg
>>76926
Смотря какая версия теории множеств. Например, в одной из них есть такая аксиома.
Литература: Пол Дж. Коэн, Теория множеств и континуум-гипотеза, Москва, "Мир", 1969.
>> No.76979 Reply
File: 1332685562374.jpg
Jpg, 2015.04 KB, 4000×2667 - Click the image to expand
edit Find source with google Find source with iqdb
1332685562374.jpg
Котаны, а щито делать если хочется вспомнить ВЕСЬ школьный курс математики. То есть с 1 по 9 класс. Не понимаю с какого боку к этому подойти.
Подскажите как быть. Гуглил "Математика с нуля" и прочую хуету. Ничего нормального не нашёл. Надавайте советов мудрых.
С меня няшка.
>> No.76980 Reply
Листочком по анализу никто не поделится?
>> No.76983 Reply
Если я раздобуду психостимуляторы, то стану ли я лучше вникать в матан? А если и буду, то будут ли доступны результаты ботания в нормальном состоянии или только под адероллом? Влияют ли подобные в-ва на понятливость или только помогают в механической зубрёжке?
>> No.76984 Reply
>>76980
Завтра отсканю.
Там ничего интересного.
Фихтенгольц-с.
>> No.76985 Reply
>>76983
Они помогают понять быстрее то, что при прочих условиях ты понял бы медленно.
Они не помогают тебе понять того, чего ты понять не можешь.
>> No.76986 Reply
>>76979
Как можно забыть школьный курс математики?
Офисня штоле?
>> No.76987 Reply
>>76979
>>74774
На рутрекере выложены учебники по алгебре с 7 по 11 класс. Качай, наливай чай, читай.
>> No.76988 Reply
File: 35788-173151-3ae59af059870a8dae1a443063ed878e.jpg
Jpg, 23.66 KB, 448×326 - Click the image to expand
edit Find source with google Find source with iqdb
35788-173151-3ae59af059870a8dae1a443063ed878e.jpg
>>76986
В школах учат ерунде, которая моментально забывается.
>> No.76989 Reply
>>76988
> В школах учат ерунде
Так вот я как раз про тоже.
Непонятно, что там можно забыть вообще.
Всю школьную программу по "математике" можно изложить на двух листах, и будет понятно.
>> No.76990 Reply
File: 463f49796401df6c0985a7abbd4ac64b.jpg
Jpg, 496.60 KB, 1000×1059 - Click the image to expand
edit Find source with google Find source with iqdb
463f49796401df6c0985a7abbd4ac64b.jpg
>>76989
Очевидно, практические навыки вычисления, смысл синуса-косинуса-тангенса-котангенса-секанса-косеканса-радикала-логарифма-интеграла, доказательство теорем Пифагора, синусов и косинусов, планиформулы для вписанных-описанных непотребств, предел последовательности как "точка сгущения" и так далее.
>> No.76991 Reply
File: 11945965_2667400_00474400a0.jpg
Jpg, 24.11 KB, 354×373 - Click the image to expand
edit Find source with google Find source with iqdb
11945965_2667400_00474400a0.jpg
>>76928
Удваиваю.
>> No.77030 Reply
>>76928
HALP, завтра утром экзамен.
>> No.77032 Reply
>>76975
Окай, непременно ознакомлюсь.
Покамест, на всякий случай уточню вопрос: вот есть у нас аксиома объединения, которая сформулирована для "какое-то множество" и "какое-то еще множество", объединение которвых существует. Но меня беспокоит что-то вроде зеноновой быстрочерепахи со слоуахиллесом - если у нас множеств бесконечно много - то почему мы можем утверждать, что их объединение таки существует? Ведь смысл бесконечности, если я ничего не напутал, как раз в том, что сколько бы мы множеств не объединили - всегджа найдется такое, которое мы еще не успели приклеить.
>> No.77033 Reply
File: 1285830955120.jpg
Jpg, 1206.96 KB, 2442×2615 - Click the image to expand
edit Find source with google Find source with iqdb
1285830955120.jpg
>>77030
Ну помогите же людям кто-нибудь, коллеги. Я не умею в это.
>> No.77040 Reply
>>77030
Ок. Попроси помощи на dxdy.ru, няша. Я не в теме, а остальные аноны, похоже, окуклились.
>> No.77046 Reply
File: DSC00274.JPG
Jpg, 67.34 KB, 1024×703 - Click the image to expand
edit Find source with google Find source with iqdb
DSC00274.JPG
>>77033
Окей, брате, мне помогли и я прошарил почти все. Ориджинал контент рилейтед. Если не понятно - спрашивай.

Единственное что я не понял - почему в системе с Сшками выбран x0, а не любой другой.
>> No.77047 Reply
>>77046
Ух ты. Спасибо, буду разбираться. Ты няша.
>> No.77050 Reply
>>77046

Что это вообще за "численное дифференцирование"? Я раньше никогда не слышал о таком.
Почему система для коэффициентов c_i несовместна?

Что значит f'(x)|x0 = c0f(x0) + c1f(x1) + c2f(x2)? Откуда взялись x1 и x_2?
>> No.77051 Reply
>>77050
Гугли "численные методы".
>> No.77052 Reply
Матаны, а я вот просто мимо проходил и неистово захотел пожелать всем ДОБРА.
>> No.77054 Reply
>>77050
> Что это вообще за "численное дифференцирование"?
Ну здрасте, вычмат же. Это когда функция задана в виде таблицы и тебе надо найти значения ее производной в произвольной точке.
> Что значит f'(x)|x0 = c0f(x0) + c1f(x1) + c2f(x2)?
Метод заключается в том, что мы полагаем, что значение производной в точке х0 можно представить в виде вот такой суммы (с погрешностью, само собой).
Значения функции в точкаx xi мы знаем, надо определить коэффициенты. Они определяются таким образом, как показано на пикрелейтеде.

Короче говоря, в конкретной задаче все xi и f(xi) известны.
>> No.77055 Reply
>>76888
> программист старший
> образование ПТУ
Хуёво быть тобой.
> да нестандартно мыслить
Ну да, конечно. Рабочий Вася тоже нестандартно мыслит, когда внезапно не берёт поллитра после работы.
> хочу заниматься Computer Science
> образование ПТУ
Подлива пролилась. Няша, люди не просто так учатся по 4 6-9 лет. На самом деле, то что рассказывают в универе на баке - ненужная по большей части хуйня. Но вместе с тем, ты так или иначе становишься математически грамотным, появляется база для правильного мышления (поэтому гораздо охотнее берут людей с высшим образованием, если вуз худо-бедно престижный, это неплохая гарантия, что человек сходу разберётся с техдоками и начнёт успешно вкатываться в процесс).
Самому разобраться с математической программой будет очень трудно, потому что математика - наука комплексная, в ней очень много ответвлений, которые, тем не менее, между собой связаны, а причинно-следственные свзяи раскрываются именно при непосредственном обучении.
Поэтому твой выход - идти на 1 курс, что все нормальные люди начинают делать лет в 16-17. Просто потому, что наука - это не сайтики клепать. Если ты, конечно, серьёзен.
мимоаспирантлиспоблядь
>> No.77056 Reply
>>77054

Ок. Но почему система получается несовместной?
(Из второго уравнения следует, что 0=1)
>> No.77057 Reply
>>77056
Тот товарищ, видимо, неправильно раскрыл скобки. У меня другая система получилась, совместная.
>> No.77058 Reply
File: DSC00272.JPG
Jpg, 68.80 KB, 1024×729 - Click the image to expand
edit Find source with google Find source with iqdb
DSC00272.JPG
Запощу и другой листок, содержащий описание этой фигни в общем виде.
Я так и не вкурил откуда в системе вылезает х0. Почему 0, а не 1?
>> No.77066 Reply
>>77046
Во, другой брат пояснил насчет x0 в системе. На самом деле коэффициенты ci зависят от x, а в систему вместо x0 надо засовывать просто x.
В результате мы получим решения для каждого ci, и эти решения будут содержать x в себе.
После подстановки этих ci в производную они посократятся и получится просто функция с одной переменной.
>> No.77075 Reply
File: Scanitto_2012-09-14_001.jpg
Jpg, 2392.24 KB, 2348×3278 - Click the image to expand
edit Find source with google Find source with iqdb
Scanitto_2012-09-14_001.jpg
>>77046
>>77066

Всё, наверное, вот так должно выглядеть.
>> No.77076 Reply
>>77055
Да, хуево быть мной. Хотя нет, не настолько. У меня есть напор, я думаю. А так же думаю у меня все получится. Как и получилось то, к чему стремился в своем мухосаранске.
Ну и не все считают так как ты. Поэтому я не буду ГРУСТИТЬ, ведь это всего лишь мнение.
Алсо, клепать сайтики - работа для петухов. Я говорю о ПРОГРАММИРОВАНИИ.
>> No.77078 Reply
>>77076
Ну и последняя отмазка. Я конечно же хотел бы стать тру математиком. Но для меня, программиста, это скорее всего уже непостижимо. Ибо выбрал другой путь. Поэтому мне достаточно тех основ для программистов что пригодятся на практике. Спасибо за внимание.
Алсо, что скажут математикобоги.Есть ли шанс попробовать себя в каком-нибудь НМУ спустя лет, например, двух самостоятельного изучения математики дома ну и с братишками-математиками/репетиторами?
>> No.77084 Reply
>>77055
> поэтому гораздо охотнее берут людей с высшим образованием, если вуз худо-бедно престижный, это неплохая гарантия, что человек сходу разберётся с техдоками и начнёт успешно вкатываться в процесс
бычье дерьмо, если ты понимаешь о чем я. Охотнее берут людей готовых обучаться и заинтересованных в деле, а не с ВО. В большинстве случаев они получают ВО чтобы просто было. Не раз собеседовал молодежь из Бауманки которая просто сосала без всяких шансов. Но часто попадались люди без ВО по которым было видно что они ШАРЯТ, есть смысл с ними работать дальше. В общем раз на раз не приходится. Надо чтобы человек был заинтересован. С ВО или без внимания не имеет.
Я имею в виду конкретно программирование
>> No.77085 Reply
>>77084
> внимания
значения, простите
>> No.77086 Reply
File: 1347570219883.png
Png, 0.95 KB, 300×20 - Click the image to expand
edit Find source with google Find source with iqdb
1347570219883.png
>>77078
Имо, если заниматься в течение двух лет математикой, то НМУ не нужен. Ты уже перерастёшь этот уровень, и если и будешь тянуть лямку ради диплома, то будешь чувствовать, что учат чуши.
>>77032
Объединение происходит по произвольному множеству, условие двухэлементности которого не ставится. На пикче из >>76975 множество, объединение элементов которого рассматривается, обозначено через x, объединение обозначено через y (я думаю, x и y не переменные теории, а переменные метатеории для переменных теории, то есть записана не аксиома, а форма записи аксиомы, аксиому из которой можно получить подстановкой вместо x, y, z конкретных переменных теории).
> если у нас множеств бесконечно много - то почему мы можем утверждать, что их объединение таки существует?
По аксиоме.
>> No.77089 Reply
>>77032
Дополню свой ответ >>77086.
> если у нас множеств бесконечно много - то почему мы можем утверждать, что их объединение таки существует?
Внутри теории по аксиоме объединения. Если же ты спрашиваешь о более глубоких мотивах, когда на теорию смотрят со стороны с мыслью "какие бы нелогические аксиомы добавить, чтобы теория была достаточно удобной?", то они мне неизвестны.
>> No.77163 Reply
Почему такой пиздец на семинарах на первом курсе в НМУ?
>> No.77164 Reply
Пусть тред кто-нибудь новый запилит. И обязательно вставит список учебных материалов, хотя бы самых попсовых учебников, чтобы вопросы повторялись пореже.
>> No.77165 Reply
>>77163
В каком смысле пиздец? Народу наверное много.
Отсканте по алгебре и анализу первые листочки пожалуйста, если есть у кого.
>> No.77166 Reply
>>77164
http://lj.rossia.org/community/studium/1717.html#cutid1 вот список, ничего лучше здесь не придумают
>> No.77171 Reply
>>77163
Поддерживаю. Сегодня был пиздец. Хорошо, что (как я понял) решения можно на почту высылать.
На геометрии всё как раз было относительно прилично.
>> No.77237 Reply
Господа, таки кто-нибудь уже запилит новый тред со списком книг?
Ньюфаг
>> No.77250 Reply
File: 1347730314202.png
Png, 0.81 KB, 300×20 - Click the image to expand
edit Find source with google Find source with iqdb
1347730314202.png
>> No.77253 Reply
>>77250
я понял, меня интересует сам тред, не очень комфортно же
>> No.77403 Reply
>>77086
> Объединение происходит по произвольному множеству
Я правильно понимаю, это произвольное множество - из тех сущностей, что называются "семействами"?
> Если же ты спрашиваешь о более глубоких мотивах
Да нет, я всего-то неспеша пытаюсь разобраться, что же на самом деле гласят аксиомы.
>> No.77409 Reply
>>77086
Если что - "бесконечно много" не обязательно значит "все".
Потому их объединение вполне может быть множеством, ZF это не противоречит.
Когда надо объеденить в одну сущность что-то, что "все X", такую сущность называют категорией (классом).
>> No.77417 Reply
File: 1347817819587.png
Png, 1.04 KB, 300×20 - Click the image to expand
edit Find source with google Find source with iqdb
1347817819587.png
Следующий тред открыт здесь: >>77310.
Капча велит разуваться перед тем, как входить: "Жива(о) разувайся!".
>> No.77492 Reply
>>77403
Множества кое-кто называет семействами, классами, и т.п. О теории семейств никогда не слышал. Множество, будь оно произвольным или нет, в рамках ЗФ подчиняется аксиомам ЗФ.
Аксиома объединени:
для любого множества y существует множество x, такое, что для любого z z принадлежит x тогда и только тогда, когда существует w, принадлежащее y, такое, что z принадлежит w.
То есть, зная, что у - множество, мы можем, по аксиоме, утверждать, что нечто, содержащее все элементы элементов множества у, тоже является множеством.
>> No.78387 Reply
Вечер, матемач. Помоги пожалуйста разобраться.
Пытаюсь доказать, что ограниченное множество в эрэн с евклидовой метрикой компактно. Делаю так:
Пусть множество ограничено, под этим понимаем, что есть такой шар, в который влезает все множество.
Тогда предположим, что наше множество не компактно, а стало быть, какую последовательность его точек ни возьми - из нее не получится набрать фундаментальную. Значит, расстояние между любыми двумя точками этой последовательности никак не меньше некоторого ε. Возьмем и запилимс открытый шар вокруг каждой точки последовательности радиусом ε/2.
Интуитивно - вот оно, противоречие - внутри шара конечного радиуса уместилось бесконечно много непересекающихся шаров радиуса ε/2.
Теперь вопрос: а туда ли я вообще зашел в рассуждениях? И если да - то как не привлекая меру строго показать противоречие?
>> No.79044 Reply
Тем времененм новый тред: http://dobrochan.ru/u/res/77310.xhtml

Петушки, не могли сразу вбросить ссылку? А я то думал, почему перестали писать?
>> No.79328 Reply
Сап, доброаноны. Помогите школьнику решить, иначе смерть меня ждет.

lim n^2-2n-3/n^2-5n+6
n→3

lim n^2-1/5n^2+4n-1
n→-1

lim n^6-1/n^3-1
n→1
>> No.80292 Reply
А подкиньте годной книженции по Банаховым алгебрам.
>> No.80773 Reply
File: shot_065.jpg
Jpg, 494.57 KB, 1920×1080
edit Find source with google Find source with iqdb
shot_065.jpg
File: shot_066.jpg
Jpg, 569.65 KB, 1920×1080
edit Find source with google Find source with iqdb
shot_066.jpg

Матаны, есть два скриншота. На них разные круги разлёта снарядов (зелёные штрихи). Пикселей на 10 разница, но она есть. 66й - с навыком наводчика (100+10)%. 65й - с навыком (100+5+11+5)%. Может кто-нибудь как-то сравнить их радиусы и придумать какую-нибудь формулу?
>> No.80836 Reply
>>80773
Танкоблядь - не человек. Сажи тебе.
>> No.80839 Reply
>>80836
Не осилишь - так и скажи. Бравировать комплексами за пределами эмо-тусовки не катит.
>> No.81084 Reply
Здравствуйте. Подскажите, пожалуйста, как доказать, что разность оснований любой трапеции больше разности её боковых сторон?
>> No.81278 Reply
>>81084
Лол, насколько я помню геометрию 8 класса, разность боковых сторон всегда равна нулю, а разность оснований строго больше нуля. По определению. Ибо боковые стороны равны, а основания не равны.
>> No.81315 Reply
>>81278
> Ибо боковые стороны равны
Но ведь это только в равнобедренной трапеции.
>> No.81352 Reply
Анон, помоги, пожалуйста, доказать существование предела последовательности log5(n)/sqrt(n) (логарифм от n по основанию 5 делить на квадратный корень из n).
>> No.82681 Reply
File: RY6Lpq9YQQA.jpg
Jpg, 63.09 KB, 604×453
edit Find source with google Find source with iqdb
RY6Lpq9YQQA.jpg
File: 9TfrokdUIjo.jpg
Jpg, 60.17 KB, 604×453
edit Find source with google Find source with iqdb
9TfrokdUIjo.jpg
File: oYdHJsZ_Efg.jpg
Jpg, 48.61 KB, 604×453
edit Find source with google Find source with iqdb
oYdHJsZ_Efg.jpg

Сап, добрач.
Так вышло, что мне на завтра нужно сделать расчетку по матану, а узнал я об этом только что. И все бы хорошо, если бы последние пару месяцев я бы ходил в универ, а не просиживал возле отца в больнице. Очень надеюсь на вашу помощь. В семье беда, сейчас пришлось работать много, и постоянно находиться возле отца, а если еще и потом из универа отчислят и лишат стипендии, будет совсем плохо. Пикрилейтед, перевод тоже.
>> No.82683 Reply
>>82681
1:
10. Доказать разницу множеств.
11. Указать, в каком соотношении находятся множества.
12. Установить взаимно однозначное соответствие.
4,4. Определить мощность.
6,7. Найти и определить, определить достижение.
10,11. Доказать справедливость утверждения.

2:
4. Множество всех комплексных чисел.
5. Множество всех корней уравнения, где ... — дробная часть.
6. Множество рациональных решений неравенства.

3:
Примечание. В некоторых примерах необходимо воспользоваться таким неравенством:
>> No.82684 Reply
>>82681
1:
10. Доказать разницу множеств.
11. Указать, в каком соотношении находятся множества.
12. Установить взаимно однозначное соответствие.
4,4. Определить мощность.
6,7. Найти и определить, определить достижение.
10,11. Доказать справедливость утверждения.

2:
4. Множество всех комплексных чисел.
5. Множество всех корней уравнения, где ... — дробная часть.
6. Множество рациональных решений неравенства.

3:
Примечание. В некоторых примерах необходимо воспользоваться таким неравенством:


Password:

[ /tv/ /rf/ /vg/ /a/ /b/ /u/ /bo/ /fur/ /to/ /dt/ /cp/ /oe/ /bg/ /ve/ /r/ /mad/ /d/ /mu/ /cr/ /di/ /sw/ /hr/ /wh/ /lor/ /s/ /hau/ /slow/ /gf/ /vn/ /w/ /ma/ /azu/ /wn/ ] [ Main | Settings | Bookmarks | Music Player ]